Virginia Questions Flashcards

1
Q

A 50 year old female with a ten year history of type II diabetes presents for regularly-scheduled follow up. She has no complaints, and just visited her ophthalmologist last week. Current medications include glyburide, metformin, and simvastatin. On physical exam, vital signs are virtually unchanged from previous visits, with temperature 37.1 C (99 F), HR 80, BP 140/83, RR 15, and O2 Sat 98% on room air. Neurological examination reveals diminished sensation to light touch and pinprick in a stocking distribution on the lower extremities bilaterally. Remainder of physical exam is benign. Laboratory evaluation reveals: Na+ 136, K+ 3.9 Cl- 104, HCO3- 25, BUN 15, Cr 1.0, Glucose 150; hemoglobin A1c: 7.1%; Urinalysis: negative for ketones, glucose, bilirubin, leukocyte esterase, or blood; moderate protein; Lipid profile: Total cholesterol 146, HDL 46, LDL 100. At this time, which of the following would be the most appropriate intervention?

A.Increase simvastatin

B.Increase glyburide

C.Increase metformin

D.Add hydrochlorothiazide

E.Add lisinopril

A

E.Add lisinopril

The patient’s lipid profile is technically at her target LDL – thus, she does not need an increase in her statin (answer A).

Key teaching point: All diabetics should be on an ACE inhibitor or ARB for cardiovascular and renal protection.

As an additional teaching point, every health maintenance visit with a diabetic patient should focus on the “Diabetic Five” – these five things, in this order: 1) Smoking cessation 2) Blood pressure control 3) Lipid control 4) Aspirin/metformin 5) Glucose control

How well did you know this?
1
Not at all
2
3
4
5
Perfectly
2
Q

A 55 year old female comes to the emergency department complaining of a headache for the past six hours. Her headache began abruptly after she finished eating breakfast, and quickly increased to 8/10 throbbing pain located mainly over her right temple. The pain has been associated with mild nausea but no vomiting. She denies chronic or recurrent headaches, but did have one headache similar to this one two weeks ago, which resolved after taking ibuprofen and lying in a quiet, dark room. She has smoked one pack of cigarettes daily for 38 years. On physical exam, the patient has temperature of 37.0 C (98.6 F), pulse of 99, and BP 147/95. Neurological examination is nonfocal, but mild photophobia and nuchal rigidity are noted. Fundoscopic examination reveals no papilledema. Skin exam shows no lesions. CT of the head, obtained without contrast, reveals no abnormalities. What is the most appropriate next step in the management of this patient?

A.Obtain head CT with contrast

B.Lumbar puncture

C.Administer i.m. sumatriptan

D.Administer oral ibuprofen

E.Administer i.v. ceftriaxone

A

B.Lumbar puncture

E.

Ceftriaxone (answer E) is the most commonly used empiric antibiotic for adults with suspected bacterial meningitis since it penetrates the CSF and covers the most likely pathogens in this age group: S. pneumoniae (60%) and N. meningitides (20%). This patient is afebrile, however, and her stiff neck and photophobia are simply signs of meningeal irritation – which in this case is caused by SAH, not a bacterial infection. Meningitis may cause headache, but not as suddenly as the headache described in the question. Even if this patient was suspected to have meningitis, you would likely want to obtain an LP before giving antibiotics so that you don’t sterilize the cultures! After obtaining an LP, antibiotic therapy with ceftriaxone (or cefotaxime) and vancomycin could be started. (The latter is used to cover the possibility of drug-resistant S. pneumoniae while you await results of the CSF culture.)

How well did you know this?
1
Not at all
2
3
4
5
Perfectly
3
Q

A four week old male infant is brought by his mother to the physician following one week of emesis. The patient’s mother states that the patient has been vomiting non-bilious material immediately after each feeding, but then becomes fussy and demands to be fed again. She denies ever seeing any blood in the emesis. Over the past two or three days, the infant’s vomiting has become increasingly sudden and forceful. The child is irritable, with few tears. The oropharynx is dry, the infant’s fontanelles appear sunken, and moderate skin tenting is noted. Capillary refill is approximately 2 seconds. On abdominal exam, visible peristaltic waves are observed, and a 1cm firm mass is palpated in the right upper quadrant. What is the most likely laboratory finding?

A.Na+ 130, K+ 2.9, Cl- 89, HCO3- 35

B.Na+ 138, K+ 3.8, Cl- 100, HCO3- 26

C.Na+ 150, K+ 4.0, Cl- 100, HCO3- 24

D.Na+ 140, K+ 3.8, Cl- 100, HCO3- 15

E.Na+ 130, K+ 5.8, Cl- 110, HCO3- 20

A

A.Na+ 130, K+ 2.9, Cl- 89, HCO3- 35

Repetitive vomiting of acidic gastric juices causes the loss of HCl – thus, the patient’s Cl- should be low. Since you’re losing H+, there will also be a metabolic alkalosis, so the HCO3- will be elevated as well. Finally, in order to maintain pH balance, the kidneys avidly reabsorb H+, but they can only do this at the expense of K+, resulting in hypokalemia.

How well did you know this?
1
Not at all
2
3
4
5
Perfectly
4
Q

A 30 year old female presents to her physician with a breast mass. She first noted a small “lump” in her left breast while showering about six weeks ago. She has noted no change in the size of the mass since that time, and she denies pain or nipple discharge. Family history is significant for a paternal grandmother who had breast cancer at age 79. Physical examination reveals a soft, round, mobile 1cm mass in the lower outer quadrant of the left breast. No skin changes are noted. What is the most appropriate next step in the management of this patient?

A.Mammography

B.Refer the patient for radical mastectomy

C.Begin levonorgestrel/etinyl estradiol

D.Genetic testing for BRCA1 and BRCA2

E.Ultrasound of breast mass

A

E.Ultrasound of breast mass

Key teaching point: Mammogram is the preferred imaging study for women over 35, while women younger than 35 should get ultrasound to evaluate a breast mass. In women younger than 35, the breast tissue is often too dense to evaluate mammographically, and the incidence of breast cancer younger women is still very low.

How well did you know this?
1
Not at all
2
3
4
5
Perfectly
5
Q

An otherwise healthy 8 year old girl presents with two weeks of perianal pruritis. She has two younger brothers, one of whom has had similar complaints for the past few days. Physical exam reveals perianal erythema with mild excoriations. The “scotch tape test” reveals several bean-shaped white eggs. What is the most likely diagnosis in this patient?

A.Trichuriasis

B.Enterobiasis

C.Child abuse

D.Fecal soilage

E.Atopic dermatitis

A

B.Enterobiasis

Whipworm or trichuriasis (choice A) is a common intestinal helminthic infection worldwide, with the highest prevalence in tropical regions. Hosts are usually asymptomatic, though the disease can cause loose stools that contain mucus or blood, resulting in a secondary anemia. Trichuriasis is also classically associated with rectal prolapse in a patient with a heavy parasite load. Heavy loads can affect a child’s growth and cognition. Diagnosis is made by stool examination for eggs, which are barrel shaped with a hyaline plug at each end.

This is a classic case of enterobiasis (answer B) or “pinworm.” The most common presenting symptom is intense anal itching or pruritus ani. Other symptoms (such as abdominal pain/fullness or nausea and vomiting) may occur if the worm burden is high.

How well did you know this?
1
Not at all
2
3
4
5
Perfectly
6
Q

A seven year old female is brought to the physician by her mother because of facial swelling and dark, cola-colored urine. These symptoms began abruptly two days ago and have been associated with anorexia and malaise. There have been no known sick contacts. Her mother states that the child is up to date with her immunizations and has been in good health except for a runny nose and sore throat around two weeks ago, which resolved after a few days without treatment. Vital signs are temperature 37.2 C (98.9 F), heart rate 95, and blood pressure of 148/86. There is diffuse edema of the lower extremities, face, and eyelids. Lungs and heart are clear to auscultation. Urinalysis shows moderate hematuria and proteinuria, and dysmorphic RBCs and occasional RBC casts are noted on microscopic examination. Based on these findings, what is the most likely diagnosis?

A.IgA nephropathy

B.Alport syndrome

C.Thin basement membrane nephropathy

D.Postinfectious glomerulonephritis

E.Henoch-Schonlein purpura

A

D.PSGN

the key features of post-streptococcal GN were all there. These include the patient’s age (<7 years old), dark brown colored urine (representing hematuria), and periorbital and peripheral edema. There is also a latent period of around 10 days following pharyngitis before symptoms of glomerulonephritis occur, although for glomerulonephritis following streptococcal impetigo, the latent period can be as long as 3-4 weeks. One diagnostic finding not mentioned was the anti-streptolysin O titer, which you would expect to be positive (indicating recent exposure to Group A strep). Finally, the finding of RBC casts is an important one. When you see casts in the urine sediment - whether WBC, RBC, or granular - you have glomerular disease.

How well did you know this?
1
Not at all
2
3
4
5
Perfectly
7
Q

A 30 year old female comes to her physician’s office for a routine health examination. She has been in good health recently and is up to date with her gynecological examinations. Her only medication is loratadine for seasonal allergies and ibuprofen for occasional headaches. Physical examination shows temperature 37.1 C (98.8 F), pulse 80, BP 170/92, RR 14, oxygen saturation of 99% on room air. A 3/6 mid-systolic ejection murmur is present. Abdomen is nontender with a soft systolic-diastolic bruit that lateralizes to the left side. The patient is grossly intact neurologically, and fundoscopic examination shows sharp optic disc margins. Laboratory evaluation shows Na+ 141, K+ 3.9, Cl- 106, HCO3- 27, BUN 18, Cr 1.0, glucose 98. Urinalysis shows trace proteinuria and no casts. What is the most likely pathological mechanism causing this patient’s hypertension?

A.Fibromuscular dysplasia

B.Oversecretion of aldosterone

C.Exogenous administration of corticosteroids

D.Atherosclerotic disease

E.Catecholamine-producing tumor

A

A.Fibromuscular dysplasia

This is a case of secondary hypertension caused by renal artery stenosis. They key clues are the markedly elevated blood pressure in an otherwise healthy young person and the presence of an abdominal bruit. In a patient without vascular risk factors, the stenosis is likely to be caused by fibromuscular dysplasia of the vessel wall (answer A) instead of atherosclerotic disease. Two quick teaching points: 1) This patient is a textbook example of fibromuscular dysplasia, which typically occurs in females under 50 years old. To confirm the diagnosis requires imaging – typically renal ultrasound with Doppler or CT or MR angiography. 2) Although the vast majority of patients with hypertension have essential hypertension, elevated blood pressure can be a sign of a more serious underlying disorder. You should always consider the diagnosis of secondary hypertension in patients who have a known onset of hypertension before age 30, patients whose blood pressure remains elevated despite multiple medications, patients who have severe (>160/100 mm Hg) hypertension above age 55, or patients with a sudden increase in their blood pressure from a stable baseline.

Atherosclerotic disease (answer D) is certainly a cause of renal artery stenosis, and would explain her abdominal bruit. The tip off in this question was that the patient was a young, previously-healthy woman - just the kind of patient who gets fibromuscular dysplasia. Atherosclerotic plaques are more likely to cause stenosis in older patients with vascular risk factors such as smoking, diabetes, or hyperlipidemia.

How well did you know this?
1
Not at all
2
3
4
5
Perfectly
8
Q

A 23 year old male presents with syncope. He reports that while walking briskly to his car, he felt his heart “racing” in his chest, and shortly thereafter passed out. The patient denies any prior syncopal episodes, but does note occasional episodes of palpitations that occur after moderate activity or during periods of increased stress. There is no family history of neurological disease, cardiac disease, or sudden cardiac death. On physical examination, pulse is 85 and regular, BP is 124/74, respiratory rate is 16, and oxygen saturation is 98% on room air. Cardiac examination reveals pulsation at the fifth intercostal space at the left parasternal area in the midclavicular line. S1 is within normal limits, and S2 is heard to split on inspiration. The remainder of the physical exam is unremarkable. EKG taken in the office shows the following tracing in lead II. Which of the following would be the most appropriate treatment for this patient’s disorder?

A.Radiofrequency ablation of pre-excitation pathway

B.Urgent DC cardioversion

C.Heart transplant

D.Biventricular pacemaker placement

E.Coronary angioplasty

A

A.Radiofrequency ablation of pre-excitation pathway

This question describes a classic presentation of Wolff-Parkinson-White (WPW) syndrome, a rare disease that is frequently tested on the USMLE. This disorder is caused by the presence of an “accessory pathway,” an abnormal conduction pathway that allows electrical impulses from the SA node to reach the ventricle without having to conduct through the AV node. Normal conduction through the AV node is slower than conduction through this accessory or “pre-excitation” pathway, so the ventricle gets activated at different times by the two different pathways. At rest, this produces the pathognomonic EKG finding of a delta wave, seen above. However, this pre-excitation pathway can lead to a number of cardiac arrhythmias, ranging from atrial fibrillation or flutter with a rapid ventricular response to ventricular tachycardia or fibrillation. Patients may experience a range of symptoms, from mild palpitations, to syncope, to sudden cardiac death. Although you may attempt to control the arrhythmias pharmacologically, the ultimate treatment for Wolff-Parkinson-White is radiofrequency ablation of the abnormal accessory conduction pathway (answer A). To answer this question correctly, you had to diagnose WPW from the EKG.

How well did you know this?
1
Not at all
2
3
4
5
Perfectly
9
Q

A 74 year old man presents with double vision. He first noticed this several months ago, and although his symptoms wax and wane, he now experiences daily episodes of “seeing double,” most frequently in the evenings. He also reports increased generalized fatigue and notes that he sometimes gets so tired at dinner that he “can hardly chew” his food. Past medical history includes osteoarthritis, hypertension, and abdominal aortic aneurysm repair. Physical examination reveals a comfortable, age-appropriate elderly gentleman with mild dysarthria. Cardiac auscultation reveals both an S4 and a 2/6 holosystolic murmur heard best at the left upper sternal border with radiation to the carotids. On neurologic exam, the patient has 5/5 strength proximally and distally. Sensation is intact and reflexes are 2+ throughout. Ocular movements are sluggish but intact in all directions. The patient has mild bilateral ptosis, which is noted to increase with sustained upward gaze. Stroking the bottom of the foot results in downward deflection of the great toe bilaterally. Which of the following is the most appropriate next step in diagnosis?

A.Administer i.v. edrophonium

B.CT of chest

C.MRI/MRA of brain and cerebral vessels

D.Temporal artery biopsy

E.CSF examination for oligoclonal bands

A

A.Administer I.V. edrophonium

This is a presentation of oculobulbar myasthenia gravis. The easiest way to confirm the diagnosis in a patient such as this one with overt ptosis is to administer i.v. edrophonium (answer A). Edrophonium is an acetylcholinesterase inhibitor with a short onset of action and half life. If you give the medication and the patient’s symptoms immediately improve, you have essentially confirmed the diagnosis. Edrophonium’s trade name is Tensilon, so you may hear neurologists talk about giving a patient the “Tensilon Test” to confirm MG.

Alternately, EMG can diagnose MG if you see a decremental response to repetitive nerve stimulation.

Here are some additional teaching points about myasthenia gravis: 1) The key feature of MG is fatigable muscle weakness. Patients will typically report that their symptoms worsen throughout the day. Oculobulbar myasthenia gravis is the most common type, resulting in the signs and symptoms presented in the question stem: double vision, ptosis, dysarthria, and difficulty chewing.

2) Myasthenia gravis has a bimodal age distribution, so there are two classic groups of patients who get MG: young women in their 20s or 30s with autoimmune disorders (RA, SLE, hyperthyroidism, etc.), and men in their 70s or 80s.
3) MG is caused by autoantibodies that bind to postsynaptic ACh receptors. A commonly tested point is distinguishing myasthenia gravis from Lambert-Eaton syndrome, which is a paraneoplastic disorder (usually associated with small cell lung cancer) in which antibodies are produced against the pre-synaptic Ca2+ channels.
4) The treatment of myasthenia gravis begins with anticholinesterase drugs like neostigmine or pyridostigmine, which increase the amount of ACh in the synapse, overcoming the antibody blockade. Prednisone or other immunosuppressive drugs are also used, and i.v. Ig or plasmaphresis are used for refractory cases to more directly target the responsible autoantibodies.
5) Myasthenia gravis almost always have some abnormality of the thymus: 75% will have thymic hyperplasia, and 15% will have an overt thymoma. Since the disease is mediated by T cells, removal of the thymus can be curative in patients who fail medical therapy. Regardless, once the diagnosis of MG has been established, it is reasonable to rule out thymoma via CT scan.

How well did you know this?
1
Not at all
2
3
4
5
Perfectly
10
Q

On routine examination, a five year old child is noted to have a loud S1 with a fixed and widely split S2 that does not vary with respiration. A soft, mid-systolic ejection murmur is heard best on the left in the second intercostal space. Remainder of physical exam is otherwise unremarkable. There is no nail clubbing, hepatomegaly, or jugular venous distension. The child is healthy and active and her mother has no health concerns. Which of the following is the most likely diagnosis in this patient?

A.Mitral valve prolapse

B.Pulmonic regurgitation

C.Tetralogy of Fallot

D.Atrial septal defect

E.Ventricular septal defect

A

D.ASD

1) ASDs, can remain asymptomatic for a long time – they are frequently picked up on routine exam. Because the pressure in the left atrium is slightly higher than the pressure in the right atrium, there is a net movement of blood from left-to-right. Over time, this overloads the pulmonary circulation and leads to pulmonary hypertension. Eventually, the pressure in the pulmonary vascular bed gets so high that the shunt reverses, causing a right-to-left shunt. This is the dreaded Eisenmenger syndrome, in which a left-to-right shunt reverses flow and turns into right-to-left shunt.
2) ASDs are silent! The murmur that is heard in this case is NOT blood flow through the ASD. Remember that to hear a murmur, you have to have turbulent blood flow. The difference in pressures between the right and left atria is so small that the blood flows easily, without turbulence. However, since there is a net left-to-right movement of blood, there is an increased volume of blood coursing across the pulmonary valve. This leads to a flow or ejection murmur heard best in the listening area for the pulmonic valve

How well did you know this?
1
Not at all
2
3
4
5
Perfectly
11
Q

A 64 year old male presents for routine health evaluation. He has been feeling well and has no complaints. Past medical history includes hypertension, osteoarthritis, and generalized anxiety disorder. Medications include hydrochlorothiazide, ibuprofen, atenolol, and paroxetine. Physical exam shows temperature 37.9 C (99.3 F), pulse 61, blood pressure 131/70, and respirations 15/min. Laboratory evaluation shows: Na+ 141; K+ 3.9; Cl- 103; HCO3- 25; BUN 18; Creatinine 1.2; WBC 9.7; Hemoglobin 10.1; Platelets 179; MCV 73 fL. Which of the following is the most appropriate next step in the management of this patient?

A.Add lisinopril

B.Measure reticulocyte count

C.Test B12 and folate levels

D.Endoscopy

E.Indirect and direct Coombs’ tests

A

D.Endoscopy

Key teaching point: Iron deficiency anemia in a male is colorectal cancer until proven otherwise. This patient presents with a microcytic anemia: his hemoglobin and MCV are both low. The differential diagnosis of microcytic anemias is fairly broad, and includes a number of “zebras.” (A useful mnemonic to remember the differential of microcytic anemias is “TAILS” – Thalassemias, Anemia of chronic disease (which can sometimes be normocytic), Iron deficiency anemia, Lead poisoning, and Sideroblastic anemia)

Far and away the top cause of a microcytic anemia is iron deficiency anemia brought about by blood loss. Any male or postmenopausal woman who presents with iron deficiency anemia should be evaluated for an occult GI source of their bleeding, preferably with upper and lower endoscopy (answer D). This patient could have an ulcer related to his NSAID use – or he could have colorectal carcinoma. Either case deserves further evaluation.

How well did you know this?
1
Not at all
2
3
4
5
Perfectly
12
Q

A 36 year old African-American male comes to the clinic complaining of fatigue and dark, cola-colored urine for the past five days. He denies fever, nausea or vomiting, recent travel, and i.v. drug use. Past medical history includes ulcerative colitis. Medications include sulfasalazine as well as trimethoprim/sulfamethoxazole which the patient began taking one week ago for a presumed Staphylococcal skin infection. Vital signs are temperature 36.8 C (98.2 F), pulse 78, blood pressure 118/72, respirations 14/min. Physical examination shows scleral icterus and a nontender abdomen without organomegaly. Laboratory evaluation shows: WBC 8.6; Hgb 9.1; Hct 27.3; Platelets 212; MCV 88 fL; Na+ 144; K+ 4.8; Cl- 101; HCO3- 26; BUN 14; Creatinine 1.0; Glucose 101; LDH 410 U/L; Haptoglobin 8 mg/dL. Which of the following is the most appropriate next step in management of this patient?

A.Quantitative IgM for hepatitis A virus

B.Begin darbopoietin injections

C.Begin methylprednisolone

D.Discontinue trimethoprim/sulfamethoxazole

E.Immediate transfusion of packed red blood cells

A

D.Discontinue TMP/SMX

This is a case of glucose-6-phosphate dehydrogenase (G6PD) deficiency, an X-linked recessive disease that tends to affect men of Mediterranean or African descent. G6PD is an enzyme in glycolysis that is responsible for the production of NADPH, which maintains glutathione in its reduced state and protects cells from oxidative stress. Since red blood cells are dependent on glycolysis and thus G6PD to produce their NADPH and glutathione, deficiency of G6PD results in RBCs that are exquisitely susceptible to oxygen radicals. Anything that increases oxidative stress results in hemolysis and an acute hemolytic anemia. The most common culprits include: viral or bacterial infections, fava beans, nitrofurantoin, quinine, dapsone, and sulfonamides. Here, the culprit was sulfamethoxazole, so that drug should be discontinued (answer D).

The disease is typically self limiting, and treatment is supportive. The key diagnostic findings in this patient are the elevated LDH and decreased haptoglobin, both of which indicate that RBCs are being lysed and are releasing their contents into the serum. The patient’s total and indirect (unconjugated) bilirubin are also likely elevated, given his clinical presentation of jaundice.

How well did you know this?
1
Not at all
2
3
4
5
Perfectly
13
Q

A 65 year old female with diabetes is found on routine screening to have a total serum cholesterol concentration of 198 mg/dL, with a serum HDL cholesterol of 58 mg/dL and serum LDL cholesterol of 128 mg/dL. Triglycerides are 78 mg/dL, and last hemoglobin A1c is 6.5%. The patient has no known history of coronary artery disease, does not smoke, and exercises daily. Current medications include rosuvastatin, fosinopril, glyburide, and metoprolol. Blood pressure is 129/78. Urinalysis shows no protein. Which of the following is the best management plan and treatment goal for this patient?

A.Add niacin to increase HDL to >80 mg/dL

B.Increase rosuvastatin to target LDL <100 mg/dL

C.Add gemfibrozil to decrease triglycerides to <70 mg/dL

D.Discontinue fosinopril and add hydrochlorothiazide to decrease blood pressure to <120/80

E.Begin insulin therapy to target Hgb A1c <6.0%

A

B.Increase rosuvastatin to target LDL <100 mg/dL

Key teaching point: Patients with diabetes are considered to have known coronary artery disease for purposes of lipid goals, and need the lowest possible LDL. This question involves lipid management. Presently, we tend to focus most of our efforts on a patient’s LDL (or “bad”) cholesterol, since there is the greatest amount of evidence linking elevated LDL to adverse cardiovascular events and death.

In general, to figure out a patient’s lipid goal, you first count up their risk factors. Cigarette smoking, a family history of premature cardiovascular disease, low “good” cholesterol (HDL <40 mg/dL), and hypertension (BP >140/90) all count as risk factors. Patients with two or more risk factors have a target LDL goal of <130 mg/dL, while patients with zero or one risk factor have an LDL goal of <160 mg/dL.

However, there’s a catch: patients with KNOWN coronary artery disease (previous MI, angina, etc.) need a lower LDL of <100 mg/dL. Also, a number of other diseases count as “coronary heart disease equivalents” and earn the same lower target LDL of less than 100. Patients with cerebrovascular disease, an abdominal aortic aneurysm, or peripheral arterial disease are assumed to have diseased coronary vessels as well. Patients with diabetes also fit into this category, so even though the patient in the question stem has no other risk factors, she still needs the most stringent LDL goal of less than 100.

How well did you know this?
1
Not at all
2
3
4
5
Perfectly
14
Q

An 84 year old woman has experienced three weeks of diffuse myalgias. In addition to worsening fatigue, she has stiffness upon awakening in her shoulders, hip girdles, neck, and torso. The stiffness usually resolves after several hours. She has also had occasional low-grade fevers and a 6 pound weight loss since her symptoms began. Physical examination shows decreased active range of motion of the shoulders and neck, but no muscle tenderness. Neurological exam shows normal sensation and reflexes. No abnormal findings are noted on skin examination. Which of the following is the most likely associated finding in this patient’s disease?

A.Anti-centromere antibodies positive at 1:640

B.Absolute neutrophil count of 950/mm3 (Normal: >1800 mm3)

C.Karyotyping showing t(9,22) chromosomal rearrangement

D.Erythrocyte sedimentation rate (ESR) of 96 mm/h (Normal: <20 mm/h)

E.X-rays demonstrating expansion of the bony cortex in a mosaic pattern

A

D.ESR of 96 mm/h (Normal: <20 mm/h)

Key teaching point: A markedly elevated ESR is the hallmark of polymyalgia rheumatica. This is polymyalgia rheumatica, one of the all-time favorites of shelf exams. Patients present with the symptoms above, though they will have few findings on physical examination. The erythrocyte sedimentation rate is usually markedly elevated (answer D) and is usually above 70-80 mm/h, but sometimes over 100 mm/h. The treatment for polymyalgia rheumatica is corticosteroids – patients will usually feel MUCH better after just a few doses of prednisone!

Another key teaching point about polymyalgia rheumatica is its association with giant cell arteritis or temporal arteritis. Patients with temporal arteritis will have temporal artery tenderness, headache, jaw pain, or evidence of ischemia (such as arm claudication or TIAs). The most ominous finding is transient visual loss, as temporal arteritis can occlude the arteries leading to the eye. This can lead to permanent blindness if the patient is not treated immediately.

How well did you know this?
1
Not at all
2
3
4
5
Perfectly
15
Q

An 18 year old female presents to her family physician to discuss options for birth control. Menarche was at age 12. Her menses occur at regular 28 day intervals, but she does have moderate abdominal cramping, bloating, and occasional nausea that sometimes prevent her from going to school. She is interested in becoming sexually active but her boyfriend, who has had other sexual partners in the past, doesn’t like to use condoms. Her family history is significant for her mother having had ovarian cancer at age 40. The patient does not smoke and is otherwise healthy. What would be the most appropriate initial step in the management of this patient?

A.Recommend condom use because of the patient’s family history of ovarian cancer

B.Prescribe an OCP after a work-up of her cyclical abdominal cramping

C.Start a progestin-only pill to limit the patient’s intake of estrogen

D.Prescribe a combined oral contraceptive

E.Recommend an intrauterine device (IUD)

A

D.Prescribe a combined oral contraceptive

1) Primary dysmenorrhea is caused by the inflammatory response (mediated by prostaglandins and leukotrienes) at menstruation that causes uterine contractions. Pain may be due to a temporary endometrial ischemic process. Combination OCPs make the endometrial lining thin, meaning that there are less inflammatory products produced at the time or menses, which in turn reduces menstrual flow and uterine contractions.
2) Questions sometimes come up regarding OCPs and cancer. Data support the assertion that OCPs help to PREVENT ovarian cancer. Currently, however, the literature does not clearly support or refute the idea that OCPs could cause breast cancer. Any patient with a family history of breast cancer should be made aware of the current uncertainty about the relationship, but advised that years of studies have shown no increased cancer risk with the more moden low-estrogen formulation pills that are prescribed today. Remember, of course, that estrogen-only supplementation should never be used in pre-menopausal women with a uterus because excessive estrogen exposure has been linked to endometrial hyperplasia and carcinoma.

How well did you know this?
1
Not at all
2
3
4
5
Perfectly
16
Q

A 55 year old woman has had four days of a cough productive of dark yellow sputum. She has also experienced occasional shaking chills and has sharp chest pain that is worst on inspiration. She denies recent travel history, known sick contacts, or recent hospitalization. Past medical history is significant for osteoarthritis and hypertension. Temperature is 38.0 C (100.4 F), pulse 85, blood pressure 132/80, respirations 22/min, oxygen saturation 97% on room air. The patient is alert, oriented, and interactive. Heartbeat is regular with normal S1 and S2. There is no JVD. There are decreased breath sounds as well as egophony and positive tactile fremitus at the right lung base. Abdomen is obese, nondistended, and nontender. There are no petechiae or rashes noted on skin exam. Which of the following is the most appropriate therapy for this patient?

A.Enoxaparin

B.Vancomycin and meropenem

C.Cefepime and gentamicin

D.Azithromycin

E.Immediate needle thoracostomy

A

D.Azithromycin

Because there is no history of the patient being hospitalized, this is community acquired pneumonia (CAP).

This patient is under age 65 with stable vital signs and no serious underlying medical illnesses – she is an ideal candidate for outpatient therapy. Once you’ve decided that a patient can be managed as an outpatient, you should select a drug regimen that covers the likely causative agents. In this setting, the most likely bug is S. pneumoniae, with the next most likely bugs being H.influenzae and Moraxella catarrhalis. You also need to consider the “atypical” causes of pneumonia like Mycoplasma, Chlamydia, and Legionella. (In fact, the CXR shown was taken from a patient with Mycoplasma.)

A significant number of S.pneumoniae strains are resistant to beta-lactams, as are many H.flu strains and almost all Moraxella strains.

Atypical pneumonias are also impervious to beta-lactams and cephalosporins, so your antibiotic choices are limited to macrolides, quinolones, and tetracyclines. Although erythromycin or tetracycline could work, macrolides like clarithromycin or azithromycin (answer D) cover H.flu better and would be the best choice among those listed.

Doxycycline or a “respiratory” fluoroquinolone like levofloxacin or moxifloxacin are reasonable outpatient monotherapies, as well.

How well did you know this?
1
Not at all
2
3
4
5
Perfectly
17
Q

A 15 year old girl is referred from her dentist after she bled excessively following extraction of her wisdom teeth. She also reports prolonged menses lasting 8-10 days and recurrent epistaxis. Her mother experienced lengthy postpartum bleeding after the delivery of all three of her children. Physical examination reveals scattered petechiae. Laboratory analysis shows: WBC 7.6; Hemoglobin 9.8; Hematocrit 29.1; Platelets 229; PT 12 s (normal 11-15s); aPTT 35 s (normal 20-35s); Bleeding time 13 min (normal 2-7 min). Which of the following is the most appropriate treatment for this patient’s disorder?

A.Plasmaphresis

B.Desmopressin

C.Factor VIII concentrate

D.Hydroxyurea

E.Splenectomy

A

B. Desmopressin

Plasmaphresis (answer A) is the treatment for thrombocytopenic thrombotic purpura or TTP. The classic pentad of TTP is microangiopathic hemolytic anemia, thrombocytopenia, mental status changes or neurological abnormalities, fever, and renal dysfunction.

This is von Willebrand’s disease, the most common hereditary bleeding disorder. This is an autosomal dominant disease that results in deficient or defective von Willebrand factor, a large glycoprotein that functions as a carrier protein for Factor VIII and also aids in platelet adhesion. To make the diagnosis, look for a positive family history and a personal history of easy bruising, mucosal bleeding, bleeding after dental procedures or tonsillectomy, or heavy menstrual bleeding. Laboratory analysis will show a normal PT, a normal or increased aPTT (depending on the degree of Factor VIII deficiency), and an increased bleeding time. The treatment for mild disease is desmopressin (answer B), which increases the production and release of von Willebrand factor from the endothelium. More severe bleeding episodes may require treatment with cryoprecipitate.

“Petechiae suggest Platelet deficiency; Cavity or joint bleeding suggests Clotting factor deficiency.”

The family history here also distinguishes between hemophilias and von Willebrand’s. Both hemophilia A (deficiency of Factor VIII) and hemophilia B (deficiency of Factor IX, also known as Christmas disease) are X linked disorders. Finding them in females would be very rare. Von Willebrand’s disease, however, is autosomal dominant, and should appear in every generation of the family regardless of sex.

Hydroxyurea (answer D) is the treatment of choice for sickle cell disease, where it increases the production of hemoglobin F. It has no role in the treatment of von Willebrand’s disease.

Splenectomy (answer E) can be used to treat refractory cases of idiopathic thrombocytopenic purpura (ITP) if initial medical treatment with drugs like corticosteroids fails. Of course, to diagnose ITP requires thrombocytopenia, and this patient’s platelet count is normal.

How well did you know this?
1
Not at all
2
3
4
5
Perfectly
18
Q

A 27 year old male has had a sore throat for two weeks. The pain has now become so great that he has great difficulty swallowing and has only been able to eat and drink small amounts over the past few days. He has had occasional fevers, but denies cough or coryza. Prior to this, the patient had been very healthy and had not been to a physician for fifteen years. Vital signs on presentation include temperature 38.0 C (100.4 F), pulse 90, blood pressure 118/75, and respirations 14/min. On physical exam, no tonsillar exudates are seen, but there are numerous soft, fluffy white plaques in the patient’s tongue and oropharynx. The lesions bleed slightly when they are scraped. Which of the following is the most appropriate next step in the management of this patient?

A.Peripheral blood smear for atypical lymphocytes

B.Rapid antigen detection test for Group A beta-hemolytic Streptococcus

C.Antibody assay for measles virus IgM (MVIgM)

D.Lateral neck radiographs

E.ELISA for HIV-1 antibodies

A

E.ELISA for HIV-1 antibodies

This is a description of candidal thrush. Fungi like candida do not typically infect normal, healthy patients. Patients who are taking immunosuppressive drugs (including steroids), who are receiving hemodialysis, or have diabetes or cancer may get candidal infections, so a good history should first seek for these factors. In their absence, you need to search out causes of acquired immunodeficiency, and the most likely of those is HIV (answer E). This patient’s history is concerning for esophageal candidiasis, so evaluation should probably also include EGD.

How well did you know this?
1
Not at all
2
3
4
5
Perfectly
19
Q

A 15 year old otherwise healthy female presents to her family physician with right lower quadrant abdominal pain. Although the pain waxes and wanes, it reaches a 7/10 on the pain scale, and ibuprofen has been ineffective. Menarche occurred at age 10 and her menses have always been regular at 26 day cycles. The patient is not sexually active. LMP was 2 weeks ago. Bimanual exam appreciates normal size ovaries, but the patient experiences pain on palpation of the right adnexae. An abdominal ultrasound reveals a 5 cm smooth, thin walled, unilocular cystic structure on the right ovary. Urine pregnancy test is negative and WBC is normal. What is the most likely diagnosis?

A.Polycystic ovarian syndrome

B.Dermoid tumor

C.Endometriosis

D.Physiological ovarian cyst

E.Tubo-ovarian abscess

A

D.Physiological ovarian cyst

Physiological cysts are usually asymptomatic initially, but as they get larger they can cause a feelings of pelvic fullness, constipation, and urinary frequency. Ultrasound is the appropriate test to confirm the diagnosis. A few additional teaching points… 1) Regarding management, if a fluid-filled cyst increases in size, is greater than 6 cm, or causes symptoms, a laparoscopic cystectomy is indicated. The cyst wall should be sent for pathological evaluation. Asymptomatic simple cysts less than 6 cm can be observed.

How well did you know this?
1
Not at all
2
3
4
5
Perfectly
20
Q

In order to obtain a job at a prison, a 34 year old man undergoes a required chest x-ray to screen for tuberculosis. A 14mm rounded opacity is noted, surrounded by normal lung tissue. There are small calcifications within the opacity, but no adenopathy or atelectasis is seen radiographically. The patient has never smoked and has no known exposures to tuberculosis. There is no family history of lung cancer or other malignancy. On physical exam, the patient has temperature 37.0 C (98.6 F), pulse 76, blood pressure 130/83, respirations 14/min. Lungs are clear to auscultation and percussion. No nail clubbing is seen. Which of the following is the most appropriate next step in the management of this patient?

A.Repeat chest x-ray in 3 months

B.Begin rifampin, isoniazid, pyrazinamide, and ethambutol

C.Bronchoscopy

D.CT guided needle biopsy

E.Thoracotomy

A

A.Repeat CXR in 3 mo

There are a number of factors that increase the likelihood that a SPN is malignant, and for a patient to be considered low risk, he or she must lack ALL of them. Having one or more of these characteristics places the individual into the moderate or high risk for malignancy and demands further workup or a tissue diagnosis.

1) Age: The risk of malignancy increases with age: only 3% of SPNs are malignant for patients 35-40, while >50% are malignant for patients over 60. Only individuals less than 35 years old are considered “low risk” for malignancy.
2) Size: Larger lesions (>3cm) are more likely malignant. Only SPNs less than 2cm can be considered low risk.
3) Radiographic appearance: Benign lesions tend to have smooth, well-circumscribed borders, while malignant nodules have irregular or spiculated borders. Calcifications are more often seen with benign lesions, although the pattern of calcification is important.
4) Rate of change: The very slowest growing lung cancers have a doubling time of around 400 days. Thus, if a lesion has been stable when compared to prior imaging for two years, it is likely benign.
5) Smoking history: Smoking is obviously far and away the #1 risk factor for lung cancer. Only non-smokers can be considered to have a low risk SPN.

In short, then, only nonsmokers under 35 years old who have an SPN less than 2cm (or one that has been radiographically stable for two years) can be considered low risk for SPN. These patients can be managed with follow up imaging, either CXR or CT, every 3-6 months initially (answer A).

How well did you know this?
1
Not at all
2
3
4
5
Perfectly
21
Q

A 7 year old girl is brought to her physician because her parents are concerned that she is shorter than all of her classmates. The patient’s past medical history is benign, and she is otherwise healthy and thriving at home and at school. Her mother is 61 in. (155 cm) and her father is 67 in. (170 cm) tall. In reviewing her old records, it appears that her growth velocity is normal. Physical examination shows the child to be in the 3rd percentile for her age for both height and weight. The patient is Tanner stage 1 and is otherwise developmentally appropriate. The remainder of physical exam is normal. What is the most appropriate next step in the management of this patient?

A.Reassurance

B.Bone age

C.HIV testing

D.Serum IGF-1

E.Serum growth hormone level

A

B. Bone age

Short stature is defined as 2 standard deviations below normal the mean height for children of the same sex and age, so this child fits the bill. The most important initial factor to consider is whether the growth velocity is normal or not. If a child has a very low growth velocity (<5th percentile) or “falls off” the growth curve, you should investigate causes for short stature like endocrinopathies, growth hormone deficiency, malnutrition or abuse, or malignancy.

the two most common causes of short stature are familial short stature and constitutional delay of growth, and both of these present with a normal growth velocity. The best test to differentiate familial short stature from constitutional delay is bone age (answer B), and that is the most appropriate next step for this patient. For familial short stature, bone age should match chronological age, while in constitutional delay, bone age will lag behind chronological age.

A couple of additional teaching points…

1) Bone age is obtained by taking radiographs of the hand and wrist, and may be described as such on the USMLE, so don’t be thrown off!
2) A child that has small stature but is overweight for their height may have growth hormone deficiency. Conversely, a child that is underweight for their height may have malnutrition.

How well did you know this?
1
Not at all
2
3
4
5
Perfectly
22
Q

A 26 year old G2P1 is at 28 weeks gestation of an uncomplicated pregnancy. The patient denies decreased fetal movement, dysuria, urinary frequency or urgency, vaginal bleeding or discharge, and nausea and vomiting. Medications include a prenatal vitamin and folate and iron supplements. Physical exam shows uterine fundal height 29 cm with fetal heart rate in the 150s. Laboratory testing shows: Fasting serum glucose 115mg/dL; Maternal blood type B+. The patient’s urinalysis shows the following. Color: yellow; Specific gravity: 1.020 (normal: 1.002 – 1.030); pH: 5.2; Glucose: negative; Ketones: negative; Protein: 1+; RBC: negative; Leukocyte esterase: trace; Nitrite: 2+; Urobilinogen: 0.3 EhrU/dL (normal: 0.2-1.0 EhrU/dL). Urine culture grows 25,000 cfu/mL Gram negative rods. Cervical cultures for N.gonnorheae and Chlamydia are negative. Which of the following is the most appropriate next step in the management of this patient?

A.Administer RhoGAM

B.Measure maternal serum alpha feto-protein (MSAFP)

C.Begin insulin therapy

D.Begin cephalexin

E.Reassurance and routine follow-up

A

D.Begin cephalexin

Key teaching point: Bacteruria of pregnancy, even if asymptomatic, should be treated with antibiotics to reduce the risk of pyelonephritis.

How well did you know this?
1
Not at all
2
3
4
5
Perfectly
23
Q

A 72 year old man comes to his physician because he has been feeling sad and “thinks he needs help.” For the past several weeks, he has noted decreased appetite and an 8 lb weight loss, and he has seldom left his house, even to participate in his weekly bowling league. While he is able to fall asleep normally, he awakens after 3-4 hours and is unable to fall asleep again. He has also had increasing difficulty balancing his checkbook and writing his Christmas cards, and believes that he just can’t stay focused enough to complete these tasks. Six months ago, his wife of fifty years died, and he often feels guilty that he did not treat her as kindly as he should have. When he thinks about his wife, he thinks that he would rather be dead than without her, although he denies having a plan to harm himself. Past medical history includes coronary artery disease and gout. Medications include lisinopril, allopurinol, metoprolol, furosemide, and aspirin. Which of the following is the most likely diagnosis in this patient?

A.Adjustment disorder

B.Bereavement

C.Major depressive disorder

D.Dysthymic disorder

A

C. Major depressive disorder

SIG: E CAPS 25 mnemonic. S – Sleep disturbance, either hypersomnia or insomnia; I – loss of Interest in usually pleasurable activities; G – feelings of Guilt or worthlessness; E – decreased Energy; C – decreased Concentration; A – Appetite disturbance, either increased or decreased; P – Psychomotor retardation; S – Suicidal ideation; 2 – symptoms must be present for at least a 2 week period; 5 – five symptoms required from the list above to make the diagnosis (and at least one of them must be either depressed mood or anhedonia).

How well did you know this?
1
Not at all
2
3
4
5
Perfectly
24
Q

A 54 year old man presents with acute knee pain which began last night while he was sleeping. He is now in 9/10 pain and says that he is hardly able to walk secondary to his inability to flex his left knee. Past medical history includes hypertension, nephrolithiasis, and two similar episodes of pain that have occurred over the past 16 months, one in his toe and another in his left knee. On physical exam, the patient is visibly uncomfortable and is holding his leg perfectly still. There is marked erythema and soft tissue swelling surrounding the knee. Passive range of motion is severely diminished secondary to the patient’s pain. Analysis of joint aspirate shows a WBC of 60,000 and abundant needle shaped crystals that show negative birefringence under a polarizing filter. Treatment is initiated with colchicine, prednisone, and indomethacin. What is the next most appropriate step in the management of this patient?

A.24 hour urine collection for uric acid

B.Measurement of serum uric acid

C.Plain film x-rays of knee

D.Begin therapy with allopurinol

E.Begin therapy with probenecid

A

A.24 hour urine collection for uric acid

Patients with gout can loosely be grouped into two categories: those who produce excessive amounts of uric acid (“overproducers”) and those who have reduced urinary excretion of uric acid (“undersecreters”). If you collect a 24 hour urine sample and find less than 600 mg of uric acid, the person is an undersecreter, and would benefit from uricosuric therapy like probenecid (answer E) or sulfinpyrazone. These medications block the reabsorption of uric acid in the renal tubule and cause increased clearance of uric acid in the urine. However, if there is greater than 600 mg of uric acid, the patient is more likely an overproducer and would likely benefit from allopurinol (answer D) to reduce the production of uric acid. Allopurinol is a xanthine oxidase inhibitor and results in decreased production of purines and uric acid. Starting either allopurinol or probenecid in an acute flare of gout is contraindicated, however, since sudden changes in the serum uric acid concentration often cause another attack.

How well did you know this?
1
Not at all
2
3
4
5
Perfectly
25
Q

A 36 year old G1P0A0 at 18 weeks gestation presents for routine prenatal care. She has no complaints and has felt the baby moving normally. Her only medication is a prenatal vitamin. Family history is significant for mental retardation in a nephew. Physical examination shows a fundal height of 17 cm and a fetal heart rate in the 160s by Doppler ultrasonography. Laboratory analysis shows alpha fetoprotein to be elevated to 3.5 multiples of the median (MoM). Unconjugated estriol and beta-hCG are within normal limits. This woman’s fetus is most likely to have which of the following conditions?

A.Fragile X syndrome

B.Cystic fibrosis

C.Anencephaly

D.Trisomy 18

E.Trisomy 21

A

C.Anencephaly

Alpha fetoprotein (AFP) is a plasma protein that is normally produced in the fetal liver. Since it crosses the placenta from the fetal circulation, measurement of the alpha fetoprotein in the maternal serum (the MSAFP) is a useful screening test for fetal abnormalities. Grossly, you can think of an increased AFP as being caused by any defect in which the fetal body cavity is left open. These include neural tube defects like anencephaly (answer C) or myelomeningocele and abdominal wall defects like gastroschisis or omphalocele

How well did you know this?
1
Not at all
2
3
4
5
Perfectly
26
Q

A 3 year old male is brought to the physician by his mother after she noted multiple bruises on his body. She denies any recent trauma. Two weeks ago, the patient had three days of cough and low-grade fever that resolved without antibiotic therapy, but since that time he has been well. Past medical history is otherwise unremarkable. The patient is up-to-date on all required immunizations. On physical exam, the patient is afebrile and playing happily at his mother’s feet in the examination room. There are no peritonsillar exudates. Tympanic membranes are clear. There is no abdominal tenderness or organomegaly. Examination of the skin shows diffuse and numerous petechiae and purpura. Laboratory analysis reveals the following: WBC 7.5 k/mcL; hemoglobin 12 g/dL; hematocrit 36%; platelets 27 k/mcL; total bilirubin 0.7 mg/dL; AST 22 U/L; ALT 18 U/L; alkaline phosphatase 120 U/L; LDH 186 U/L; PT 11 s (normal 11-13s); PTT 22 s (normal 20-30s). Which of the following is the most appropriate initial treatment for this patient’s disorder?

A.Prednisone

B.Ceftriaxone

C.Vincristine

D.Cyclophosphamide

E.Immediate platelet transfusion

A

A. Prednisone

This patient has idiopathic thrombocytopenic purpura (ITP). ITP is a diagnosis of exclusion made by finding low platelets and ruling out other diseases and toxic exposures that result in decreased platelet counts. It is an autoimmune disease caused by antibody-mediated destruction of platelets, so first line therapies are high-dose corticosteroids like prednisone (answer A) as well as intravenous immunoglobulin (IVIg). Patients refractory to these treatments may require more high-powered immunosuppressant drugs or splenectomy.

There are two groups of patients who get ITP: children from 2-4, and adults (usually women) in their 20s-40s. Children in general do much better with the disease: over 80% of children will have a spontaneous remission of their disease, while only 20% of adults have a sustained remission.

How well did you know this?
1
Not at all
2
3
4
5
Perfectly
27
Q

A 36 year old construction worker is brought to the emergency room after a house frame collapsed on him. His upper thighs were trapped under a heavy wooden beam for almost two hours while his co-workers worked to free him. Vital signs are temperature 37.2 C (99.0 F), pulse 90, blood pressure 136/82, respirations 16/min. The patient is alert and oriented, but obviously in pain. There are occasional superficial abrasions on the face and arms, but no large lacerations are noted. Breath sounds are present and equal bilaterally. There is a normal S1 and S2. Abdomen is nontender, nondistended, and atraumatic. The patient’s legs show bilateral ecchymoses and tenderness at the upper thigh. Dorsalis pedis and posterior tibial pulses are 2+, and distal leg sensation to light touch and pinprick is intact. Laboratory analysis shows the following Na+ 145; K+ 5.3; Cl- 101; bicarbonate 22; BUN 17; creatinine 1.1; glucose 106. Dipstick urinalysis is unremarkable except for 4+ blood. Microscopic urinalysis shows 2 WBCs/hpf, 0 RBCs/hpf, 3 epithelial cells/hpf, and no casts. What is the most appropriate next step in the management of this patient?

A.Intubation and mechanical ventilation

B.Bolus with 2 L i.v. 0.9% NaCl solution

C.Cystoscopy

D.Intravenous pyelogram

E.Abdominal and pelvic CT

A

B.Bolus with 2 L i.v. 0.9% NaCl solution

Key teaching point: When you find blood on a dipstick urinalysis but there are no RBCs microscopically, think of myoglobinuria. This patient has rhabdomyolysis, a common complication of crush injuries. The two most serious complications of rhabdomyolysis are renal failure induced by myoglobinuria and electrolyte abnormalities (like hyperkalemia, hypocalcemia, and metabolic acidosis) caused by the release of massive amounts of intracellular ions.

At the moment, this patient’s electrolytes do not demand immediate intervention – but the finding of large blood on urine dipstick but no RBCs on microscopic analysis shows that the patient does have myoglobinuria (myoglobin cross reacts with hemoglobin on the urine dipstick).

The most important intervention at this time is giving i.v. fluid (answer B) to prevent myoglobin-induced ATN. Patients with myoglobinuria may require massive amounts of fluid in order to maintain good urine output because of fluid sequestration at the site of injury.

How well did you know this?
1
Not at all
2
3
4
5
Perfectly
28
Q

A 37 year old teacher has had three months of non-productive cough. The cough seems to be worst at night and upon awakening in the morning. She denies fevers, dyspnea, chest pain, wheezing, purulent nasal discharge, or heartburn. Past medical history includes IBS and hypertension. Medications include candesartan and an oral contraceptive pill. She has never smoked or traveled outside of the United States. Physical examination reveals an afebrile, well-nourished, healthy appearing female. The oropharynx is moist and without erythema or exudates. Nasal mucosa is pink and slightly edematous. Breath sounds are clear to auscultation bilaterally. First and second heart sounds are within normal limits, and no additional heart sounds or murmurs are noted. There is no peripheral edema. Chest x-ray shows normal lung fields. Which of the following is the most appropriate intervention at this time?

A.Stop candesartan

B.Begin antihistamine and decongestant

C.Amoxicillin/clavulanate

D.Place tuberculin purified protein derivative (PPD)

E.Chest CT

A

B.Begin antihistamine and decongestant

This patient presents with a chronic cough, defined as any cough lasting >3 weeks. The most likely culprits in an otherwise healthy person are, in order, postnasal drip, asthma, and GERD. Since this patient has no findings that suggest GERD, asthma, or any other of the numerous other pathological causes of chronic cough, it is reasonable to treat her empirically for postnasal drip with an antihistamine and decongestant (answer B). This course of action is the most likely to both provide a diagnosis and ameliorate the patient’s symptoms. In the appropriate setting, chronic cough can be a sign of CHF or intrinsic lung disease. Smokers can have chronic cough related to emphysema, chronic bronchitis, or the irritation of the tobacco smoke itself.

How well did you know this?
1
Not at all
2
3
4
5
Perfectly
29
Q

Following a 10 hour car trip, a 73 year old female develops left ankle swelling and acute shortness of breath. Ultrasound with Doppler of her left leg confirms the presence of deep venous thrombosis, and the patient is hospitalized and started on a heparin drip and oral warfarin. Two days later she is discharged from the hospital with subcutaneous heparin and warfarin. Shortly upon arrival home, she develops pleuritic chest pain and shortness of breath and presents to her physician. Physical examination reveals tachypnea and an oxygen saturation of 89% on room air. Laboratory analysis shows her activated partial thromboplastin time to be therapeutically increased, while her prothrombin time remains below the optimal therapeutic level. A ventilation/perfusion scan is obtained, which shows a large perfusion defect in the left lower lobe. Which of the following is the most appropriate intervention at this time?

A.Hospital admission for observation

B.Begin aspirin

C.Increase heparin

D.Increase warfarin

E.Placement of an inferior vena cava filter

A

E.Placement of an inferior vena cava filter

This patient has had a pulmonary embolism while receiving anticoagulation therapy for DVT. The only intervention among those listed that is likely to help her is the placement of an inferior vena cava filter (answer E). These filters can be placed transcutaneously, and in the short term will prevent a large thrombus from making its way from her leg to her pulmonary vasculature.

30
Q

A 56 year old male with hypertension and history of drug abuse and medication noncompliance presents to the clinic for follow up. He has been feeling well since his last appointment several months ago, but does admit to cocaine use last night when he ran into some old friends. He denies any other drug use. Medications include hydrochlorothiazide, atenolol, and paroxetine. Vital signs are temperature 36.9 C (98.4 F), pulse 85, blood pressure 202/122, and respirations 14/min. Physical examination reveals a comfortable, pleasant male in no distress. There is no papilledema or retinal hemorrhage on fundoscopic exam. The patient is alert, oriented, and neurologically intact. Laboratory evaluation, including CBC, electrolytes, BUN and creatinine, and urinalysis are all unremarkable. Which of the following is the most appropriate next step in the management of this patient?

A.Obtain urine drug screen

B.Head CT without contrast

C.Administer naloxone

D.Administer oral labetalol

E.Administer intravenous nitroprusside

A

D.Administer oral labetalol

This is hypertensive urgency, defined as a systolic blood pressure >200 or a diastolic >120 in the absence of symptoms.

In contrast, a hypertensive emergency is defined as increased blood pressure with signs and symptoms of end-organ damage such as papilledema, stroke, hematuria, headache, altered mental status, acute coronary syndrome, etc.

The treatment of the two is different: for a hypertensive emergency, you need to give i.v. agents to get the blood pressure down within minutes. Hypertensive urgency, however, is treated with oral medications to bring down the blood pressure more slowly over a matter of hours. Thus, the best choice is oral labetalol (answer D).

Apart from its availability as a p.o. formulation, this patient has another compelling reason to be treated with labetalol instead of another agent: his recent history of cocaine use. Unlike other “pure” beta blockers, labetalol has both alpha- and beta-adrenergic blocking properties. If you treat a patient with cocaine-induced hypertension with a pure beta blocker, you’re left with unopposed alpha adrenergic activation, which can cause intense vasoconstriction, ischemia, and gangrene.

Administering intravenous nitroprusside (answer E) will bring the patient’s blood pressure down, but will require an inpatient (or potentially even an ICU) admission for blood pressure monitoring. If this patient were having symptoms – that is, if he were having a hypertensive emergency rather than hypertensive urgency – this would be an appropriate management plan, but in the absence of symptoms, it is a bit too much at the present moment.

31
Q

A 71 year old male with a history of coronary artery disease and congestive heart failure presents with worsening dyspnea on exertion, orthopnea, and peripheral edema over the past week. Two years ago, he had a coronary stent placed in his circumflex artery. Catheterization at that time showed an ejection fraction of 25%, and since that time the patient had been stable with medical therapy. Past medical history also includes diabetes, hypertension, and peripheral arterial disease. Current medications include clopidogrel, lisinopril, propranolol, bumetanide, spironolactone, glipizide, and metformin. Physical examination reveals diffuse crackles bilaterally extending from the lung bases to two-thirds of the way up the lung fields. An S3 is heard, and there is 3+ pitting edema of the lower extremities bilaterally. Chest x-ray shows cardiomegaly and pulmonary edema in a “bat wing” pattern. The patient is admitted to the hospital, placed on a fluid restriction, and treated with intravenous furosemide. Following a net diuresis of 3 L, the patient feels much better. Which of the following is the next most appropriate step in the management of this patient?

A.Measurement of brain natriuretic peptide (BNP)

B.Echocardiography

C.Arterial blood gas

D.Increase propranolol

E.Endomyocardial biopsy

A

B.Echo

This patient with a known history of coronary artery disease presented with the classic features of decompensated congestive heart failure. Once he has been stabilized, the focus should turn to what precipitated the worsening heart failure.

There are a number of common precipitants for CHF exacerbations, including dietary indiscretion (such as a high salt diet), medical noncompliance, new arrythmias (including atrial fibrillation), anemia, and medication changes (such as an increase in beta blockers). However, the most important cause of decompensated heart failure, and the one that should always be ruled out first, is new myocardial ischemia or infarction. An echocardiogram (answer B) will allow assessment of the patient’s ejection fraction and since we know this patient’s previous ejection fraction, we’ll know whether his coronary artery disease has progressed.

As a sidenote, another step that should be taken in the management of this patient is measurement of serum electrolytes, given his diuresis of 3 L of fluid. Because brain natriuretic peptide or BNP (answer A) increases in CHF, measuring it is useful if you are trying to determine whether CHF is the cause of a patient’s dyspnea. Given the abundance of history, physical, and radiographic findings pointing to CHF (and the absence of any cues steering you toward any other cause of dyspnea), checking a BNP is not necessary to make the diagnosis.

32
Q

A 29 year old female medical student comes to her physician with gallstones after serving as a model patient for a demonstration of abdominal ultrasound techniques. The stones were located within the gallbladder, and the patient was surprised at this finding, because she never had experienced any pain. Physical examination reveals a comfortable, afebrile, obese female. Murphy’s sign is negative. Laboratory examination shows the following: WBC 7.0 K/mm3; albumin 4.2 g/dL; Alkaline phosphatase 72 U/L; ALT 16 IU/L; AST 19 IU/L; total bilirubin 0.3 mg/dL; conjugated bilirubin 0.1 mg/dL; lipase 18 U/L (normal: 10-140 U/L). Which of the following is the most appropriate next step in the management of this patient?

A.Reassurance

B.Endoscopic retrograde cholangiopancreatography (ERCP)

C.Abdominal CT

D.Ursodeoxycholic acid

E.Cholecystectomy

A

A.Reassurance

This scenario describes an asymptomatic patient with cholelithiasis and normal LFTs. Some 10-20% of all Americans will develop gallstones, but of these, 60-80% will remain asymptomatic. This asymptomatic, otherwise healthy patient should be reassured (answer A) – other interventions can be considered should she become symptomatic in the future.

33
Q

A 38 year old HIV positive male presents for regularly-scheduled follow-up. He received pneumococcal and influenza vaccinations last year, and has had no opportunistic infections for several months. Current medications include efavirenz and nevirapine. Physical exam is unchanged from previous. Laboratory evaluation shows CD4+ lymphocyte count of 179/mm3. Tuberculin purified protein derivate (PPD) is placed intradermally, and is read 72 hours later as causing 4 mm of induration. Which of the following is the most appropriate next step in the management of this patient?

A.Chest x-ray

B.Clarithromycin

C.Trimethoprim-sulfamethoxazole

D.Fluconazole

E.Isoniazid, rifampin, and pyrizinamide

A

C.TMX-SMP

A chest x-ray (answer A) is not needed. This patient’s PPD should be read as negative, and in the absence of clinical suspicion for active TB, there is no reason to get this study right now.

Any patient with AIDS whose CD4+ count drops below 200 needs to begin prophylaxis for Pneumocystis carinii (PCP). Trimethoprim-sulfamethoxazole (answer C) is the first-line drug of choice, and should be started in this patient now.

Prophylaxis for Toxoplasma should begin when the CD4+ drops below 100. (TMP/SMZ is adequate prophylaxis for both PCP and Toxoplasma, so if the patient can tolerate it, it’s usually best to try to kill two birds with one stone.) When the CD4+ count drops below 50, you should begin azithromycin or clarithromycin for Mycobacterium avium complex (MAC) prophylaxis.

34
Q

An 18 year old G1 at 39 weeks gestation presents in labor. Her pregnancy has been complicated by excessive weight gain and gestational diabetes mellitus. Soon after presentation, the patient rapidly dilates and the infant’s head descends. The infant’s head is delivered without difficulty, but before delivering the body, the head appears to retract toward the pelvis. Initial attempts to deliver the body are unsuccessful. A fetal scalp electrode is attached, which shows a wavy tracing with heart rates between 140 and 150 bpm with marked beat-to-beat variability. Which of the following is the most appropriate next step in the management of this patient?

A.Caesarean section

B.Fundal pressure

C.Oxytocin

D.Suprapubic pressure

E.Fracturing the baby’s clavicles

A

D.Suprapubic pressure

So long as the infant is doing well, Caesarean section (answer A) is inappropriate before attempting some of the obstetrical maneuvers described above. If the situation is dire or other methods have failed, it is possible to place the infant’s head back in the pelvis and go to Caesarean section (this is called the Zavanelli maneuver).

This patient’s delivery has been complicated by shoulder dystocia, where the infant’s anterior shoulder becomes impacted behind the pubic symphysis. Often, moderate suprapubic pressure (answer D) is all that you’ll need to free the fetal shoulder, so this should be attempted first and is the best option among those listed.

The risk factors for shoulder dystocia include some of the things mentioned in the question stem, as well as anything else that causes a mismatch between the “passenger” and the “passage.” Fetal macrosomia, gestational diabetes, maternal obesity, postdate pregnancy, and prolonged second stage of labor are all identified risk factors.

35
Q

A three-week old infant is brought to the clinic after the patient’s adoptive parents noted a red rash on her hands and feet. The child was born at term via vaginal delivery to a 28 year old woman who received no prenatal care and gave her child up for adoption shortly after birth. On physical examination, the infant appears lethargic and has generalized lymphadenopathy. There is profuse, blood-tinged purulent nasal discharge, and there is an erythematous maculopapular rash on the dorsal and ventral surfaces of the hands and feet. Abdominal examination reveals hypoactive bowel sounds and hepatosplenomegaly. Which of the following is the most likely explanation of these findings?

A.Congenital toxoplasmosis

B.Congenital rubella

C.Congenital CMV infection

D.Congenital syphilis

E.Herpes neonatorum

A

D.Congenital syphilis

This is congenital syphilis (answer D). The key findings on physical exam were the rash involving the palms and soles, and the blood-tinged purulent nasal discharge (known as the “snuffles”). Lymphadenopathy and organomegaly are also characteristic.

This question brings up the issue of the TORCH infections, which is an acronym for Toxoplasmosis, Other (including HIV and syphilis), Rubella, CMV, and HSV. All of these infections can cause poor feeding, fever, deafness, mental retardation, hepatosplenomegaly, and thrombocytopenia, and so you should always consider them when you see an infant with one of those conditions. (A history of no or limited prenatal care is usually a giveaway that a question is looking for a problem – such as a TORCH infection - that would have been prevented with normal prenatal care.)

Each of the TORCH infections does have its own unique characteristics and “buzzwords” that help you distinguish it clinically from the others:

The classic triad of congenital toxoplasmosis (answer A) consists of hydrocephalus, chorioretinitis, and intracranial calcifications.

Congenital rubella (answer B) causes deafness and cataracts as well as numerous purpuric skin lesions, leading to the presentation of the so-called “blueberry muffin” baby. Congenital rubella syndrome occurs when the mother contracts rubella early on in her pregnancy – the risk of congenital rubella syndrome is very low after 20 weeks. Since the MMR vaccine contains a live attenuated virus, there is at least a theoretical risk of causing congenital rubella syndrome, and for this reason the vaccine is avoided in pregnant women.

Congenital CMV infection (answer C) is difficult to diagnose, because most of the key features (jaundice, deafness, chorioretinitis, thrombocytopenia/purpura, organomegaly) overlap so much with other TORCH infections. Acute CMV infection in adults can cause an infectious mononucleosis-like syndrome, so this may be given as a clue in the question stem.

36
Q

A 5 year old boy presents with bedwetting. His mother states that for the past two months, he has been wetting the bed several times a week. The child is embarrassed about this new incontinence, and has had no problems in the daytime. The patient’s mother insists that the child has otherwise been healthy; she denies fevers and increased appetite or fluid intake. The patient’s past medical history is remarkable only for a hospitalization at age 1 for bronchiolitis. The boy lives with his mother, father, and a younger sister who was adopted from Russia several months ago. Everyone at home has been well. On physical exam, the patient is a talkative, interactive male. Speech, motor, and physical development are all normal. What is the most appropriate initial step in the management of this patient?

A. Urine culture

B. Voiding cystourethrogram

C. Recommend an enuresis alarm

D. Desmopressin

E. Urinalysis

A

E.Urinalysis

New onset enuresis should prompt the physician to perform a thorough history and physical and to initially obtain a urinalysis (answer E). This single test will allow screening for urinary tract infection, a common cause of new-onset enuresis, as well as diabetic ketoacidosis, diabetes insipidus, and water intoxication. Imaging and referrals are reserved for patients with histories and physical exams that suggest a structural cause.

37
Q

A 56 year old man returns to the office because his calcium was found to be elevated at a previous visit. Although the remainder of his labs, including complete blood count, liver panel, and chemistries were all within normal limits, the patient’s calcium was 11.9 mg/dL (normal: 8.5-10.5 mg/dL). The patient feels well and has no complaints. Current medications include verapamil, cetirizine, and atorvastatin. EKG shows a right bundle branch block, stable from previous. Which of the following is the most appropriate initial step in the management of this patient?

A.Serum parathyroid hormone level

B.CT of chest/abdomen

C.Genetic testing for multiple endocrine neoplasia, type 1

D.Discontinue verapamil

E.Intravenous saline and furosemide

A

A.Serum parathyroid hormone level

There is a long list of things that can cause hypercalcemia, but over 80-90% of cases are caused by either malignancy or hyperparathyroidism. Of these two, the easiest to rule out is hyperparathyroidism, which is easily evaluated by drawing a serum PTH level (answer A). A few other teaching points…

1) Remember the CHIMPANZEES mnemonic for hypercalcemia? C – Calcium oversupplementation, H – Hyperparathyroidism, I – Immobility or Iatrogenic (i.e., from thiazide diuretics), M – Milk alkali syndrome, P – Paget’s disease, A – Acromegaly or Addison’s disease, N – Neoplasms, Z – Zollinger-Ellison syndrome (when it’s associated with MEN-1), E – Excess Vitamin D, E – Excess Vitamin A, S – Sarcoidosis.
2) You can remember the key signs and symptoms of hypercalcemia with the following mnemonic: “Bones, Stones, Abdominal Groans, and Psychiatric Undertones,” because the most common symptoms include bone fractures, kidney stones, vomiting and constipation, and weakness, fatigue, and altered mental status.
3) Severe hypercalcemia or hypercalcemic crisis (usually defined as the presence of severe symptoms or a calcium level over 14 mg/dL) is a medical emergency because of its tendency to cause heart conduction abnormalities. Check an EKG, and then begin therapy with i.v. fluid and furosemide. (Remember, “loops lose” calcium, while thiazide diuretics can increase calcium.)

38
Q

An 83 year old female with dementia is brought to the physician by her granddaughter because of a change in her skin color, which has taken on a golden hue over the past few days. The patient’s granddaughter also reports that she has been unable to get the patient to eat or drink for the past 24 hours, and that although the patient’s urine output has decreased, her urine appeared dark brown when she changed her diaper this morning. Vital signs are temperature 39.1 C (102.4 F), pulse 105, blood pressure 77/45, respiratory rate 24, oxygen saturation 96% on room air. On physical examination, the patient is somnolent but arousable to pain. The skin is jaundiced and the sclerae are icteric. Lungs are clear to auscultation. Abdominal examination reveals decreased bowel sounds, with diffuse tenderness to palpation. There is voluntary guarding of the right upper quadrant. Which of the following is the most appropriate next step in management for this patient?

A.Abdominal CT

B.Administer 1 L bolus of 0.9% NaCl

C.Begin i.v. nafcillin

D.Urgent endoscopic retrograde pancreaticoduodenoscopy (ERCP)

E.Percutaneous transhepatic drainage tube placement

A

B.Administer 1 L bolus of 0.9% NaCl

Key teaching point: Always, always remember the ABCs! This is acute cholangitis. The classic features of Charcot’s triad (RUQ pain, jaundice, and fever/chills) are all present, and the patient is rapidly moving toward the full Reynold’s pentad (Charcot’s triad plus shock and mental status changes). However, to answer this question correctly, you need not know the diagnosis – all you need to know are the ABCs (airway, breathing, and circulation). While this lady has a patent airway and is moving air appropriately, her blood pressure is too low. Her hypoperfusion will soon lead to end-organ damage if you do not expand her intravascular compartment, and the best way to do that is with an isotonic fluid like normal saline (0.9% NaCl, answer B) or lactated Ringer’s.

39
Q

A 35 year old man presents to his physician with neck pain that began suddenly while at rest four hours ago. His past medical history is significant for a ten-year history of disorganized schizophrenia, which has been treated unsuccessfully with a number of medications including olanzapine, risperidone, and haloperidol, and he has been recently switched to thiothixene. Examination shows a disheveled male with a flat affect, holding his head twisted to the right. There is limited cervical range of motion and pronounced muscle contraction of the sternocleidomastoid and paracervical muscles. Which of the following is the most appropriate pharmacologic intervention for this patient’s condition?

A.Dantrolene i.v.

B.Increase p.o. thiothixene

C.Add clozapine

D.Oral ibuprofen

E.Oral diphenhydramine

A

This patient has experienced an acute dystonic reaction caused by his antipsychotic medication. The so-called “typical” antipsychotics (like haloperidol, thiothixene, chlorpromazine, and fluphenazine) have powerful dopamine blocking properties, which make them ideal for treating the positive symptoms of schizophrenia like hallucinations or delusions. Blocking dopamine has the indirect effect of increasing acetylcholine, so anticholinergics that penetrate the blood-brain barrier like benztropine or diphenhydramine (answer E) are the treatments of choice for these extrapyramidal reactions.

The actions of typical antipsychotics outside of the nigrostriatal tract are collectively called extrapyramidal symptoms. These include: Akathisia – a subjective feeling of restlessness that develops several weeks after beginning medication; best treated by decreasing the antipsychotic, although benzodiazepines, beta-blockers, and anticholinergics may help. Pseudoparkinsonism or dyskinesia – includes classic Parkinsonian features such as a shuffling gait and cogwheel rigidity; best treated by withdrawing or decreasing the offending medication or adding a dopamine agonist like amantadine Tardive dyskinesia (TD) – nonsuppressable stereotypical facial and oral movements such as lip smacking. Decreasing or discontinuing the antipsychotic or adding an anticholinergic may initially make TD worse, since the disorder is thought to be due to dopaminergic receptor sensitization. Many – if not most - cases of TD are irreversible.

40
Q

A 37 year old HIV+ male presents with one week of headache and low grade fever. For the past two days, he has had nausea and vomiting. On physical exam, temperature is 38.1 C (100.6 F), pulse 80, blood pressure 108/68, respirations 15/min. The patient is somnolent but arousable. Nuchal rigidity is present. With the patient supine, passive knee extension elicits neck pain. Lungs are clear to auscultation. There are no petechiae or cutaneous hemorrhages seen. CT scan shows no focal lesion, and lumbar puncture is performed. CSF analysis shows clear, light yellow CSF with an opening pressure of 290 mm Hg (normal: 70-180 mm Hg), 25 WBC (normal 0-5) with 96% lymphocytes, 15 RBCs, a glucose of 38 mg/dL (normal: 40-70 mg/dL)and protein 50 mg/dL (normal: 15-45 mg/dL). Which of the following would be the most appropriate therapy for this patient?

A.Ceftriaxone

B.Acyclovir

C.Vancomycin and cefotaxime

D.Trimethoprim-sulfamethoxazole

E.Amphotericin B and flucytosine

A

E.Amphotericin B and flucytosine

Key teaching point: An isolated elevated opening pressure is characteristic of cryptococcal meningitis. This question describes clinical symptoms and signs of meningitis in an immunocompromised patient. Although such patients certainly can get meningitis caused by the usual bugs like Streptococcus pneumoniae and Neisseria meningitides, you also have to worry about other causes like fungus in general and Cryptococcus in particular. This patient’s LP shows lymphocytosis and a dramatically increased opening pressure, which are the key features of cryptococcal infection. Treatment of cryptococcal meningitis requires aggressive antifungal therapy with amphotericin B and flucytosine (answer E).

To confirm the diagnosis, you should order a latex agglutination test for cryptococcal antigen (CrAg) or an India ink preparation to look for encapsulated yeasts.

In real life, given the patient’s immunocompromised status, it would not be unreasonable to start empiric antibacterial agents in addition to the amphotericin/flucytosine. However, the CSF in bacterial meningitis should have a neutrophilic predominance, and the glucose should be lower and the protein should be higher than is seen in this case.

41
Q

A 68 year old woman has had worsening back pain for the past three months. Her pain radiates to her legs bilaterally. The pain is worst while walking or standing upright and is associated with weakness and numbness of her legs while walking. Sitting or lying down relieves the pain, but the patient also found that leaning forward over her grocery cart improved her symptoms while shopping earlier this morning. She has not had fever or bowel or bladder incontinence. Past medical history is significant for hypertension controlled with lisinopril. The patient does not smoke. Physical examination shows no tenderness along the lumbar spine and a negative straight-leg raising test. The feet are warm, and dorsalis pedis pulses are 2+ bilaterally. No saddle anesthesia is present, but the patient has weakness of the extensor hallucis longus bilaterally. Which of the following is the most likely pathological mechanism for this patient’s symptoms?

A.Herniation of the nucleus pulposus against the L4 nerve roots

B.Pyogenic abscess of the epidural space in the lumbar spine

C.Atherosclerotic occlusion of the popliteal arteries

D.Malignant tumor invasion of the lumbar spine

E.Narrowing of the lumbar spinal canal

A

E.Narrowing of the lumbar spinal canal

Key teaching point: An older patient with back pain that is worst while standing and walking but is relieved by sitting or spinal hyperflexion has lumbar spinal stenosis. This is lumbar spinal stenosis, caused most often by age-related degeneration of the lumbar spine and intervertebral discs leading to pathological canal narrowing (answer E). This leads to compression and ischemia of the spinal nerve roots, which in turn causes the typical presentation of “neurogenic claudication,” where symptoms occur during ambulation because of increased metabolic demands of the nerve roots. Flexing the spine increases the canal size and relieves the stenosis, and patients may describe a variety of maneuvers involving hyperflexion of the spine that they have found to relieve the pain (such as this patient leaning over her grocery cart). Although symptoms can be managed medically, the treatment of lumbar spinal stenosis is ultimately surgical.

A herniated disc compressing the nerve roots (answer A) is unlikely, since there would have to be bilateral and equal impingement to cause this patient’s symptoms. Patients with herniated discs complain of shooting, “radicular” pain. The key physical exam maneuver to diagnose nerve impingement is the straight leg raise, where the patient’s shooting pain is re-created by raising the contralateral leg. You should also seek out numbness in a dermatomal distribution corresponding to the level of the disc involved, as well as findings of motor weakness and reflex loss. If the L4 root were compressed, you would expect to find diminished knee jerk reflex (while S1 compression causes a diminished ankle jerk.) This patient’s weakness of the extensor hallucis longus demonstrates involvement of the L5 nerve roots.

42
Q

A 34 year old female presents to her family physician in tears because she wants to have a baby. She has had three second trimester miscarriages over the past five years. She and her husband, who has two children from a previous marriage, desperately want to conceive, and their failure to do so has begun to cause stress in their marriage. The patient is otherwise healthy, and there is no history of sexually-transmitted infections or hematologic or rheumatologic diseases. Menarche began at age 11 and her menses are regular at intervals of 25 days. Her pelvic exam reveals a slightly retroverted uterus and normal size ovaries. What is the most likely etiology of this patient’s recurrent pregnancy losses?

A.Maternal balanced translocation

B.Cervical incompetence

C.Paternal balanced translocation

D.Retroverted uterus

E.Amniotic band syndrome

A

This question addresses the etiology of recurrent pregnancy loss (RPL), defined as 3 or more consecutive losses of pregnancies before the 20th week of gestation. A good way to remember causes of recurrent pregnancy loss is to think about the timing of the loss. Genetic causes tend to cause early pregnancy loss, while anatomical factors tend to cause losses later in the pregnancy. An incompetent cervix (answer B) is the only anatomical cause of RPL listed and is therefore the best answer in this case.

A few teaching points…

1) The earlier the gestational age at abortion, the higher the incidence of cytogenetic defects. Abnormalities of chromosome number or structure are the most common cause of early pregnancy loss.
2) Anatomical causes of recurrent pregnancy loss tend to manifest later in pregnancy as fetal size increases. These include cervical incompetence, uterine anomalies, leiomyoma and intrauterine synechiae.
3) Immunological causes can cause pregnancy loss at any time. Antiphospholipid antibodies, for example, tend to cause both recurrent early miscarriages and repetitive late miscarriages.

43
Q

A mother brings her 8 year old son to the family physician because of strange behavior for the past year that appears to be getting worse. For the past twelve months, the patient’s mother has noted episodes of repetitive blinking or grimacing. These episodes appear to be beyond her son’s control, and often occur at inopportune times. The patient is now being teased at school by his classmates for these behaviors. During the patient interview, the child appears to be listening intently to the conversation with his mother; there is a minute or so of rapid, repetitive blinking followed by several grunting noises from the patient. When asked, the child is aware of the blinking and grunting, but cannot explain why he was doing it. The patient denies hallucinations or bizarre thoughts and states that his mood is fine. He does well in school and has a close group of friends in his neighborhood who he enjoys playing with. His growth and development are within normal limits. What is the most likely diagnosis in this patient?

A.Autism

B.Rett syndrome

C.Tourette syndrome

D.Sydenham chorea

E.Attention deficit hyperactivity disorder

A

C.Tourette syndrome

This is a classic case of Tourette syndrome (answer C), a neurological disorder characterized by motor and phonic tics with a childhood onset. Tics are defined as sudden, brief, intermittent movements or utterances that are considered to be involuntary though they can sometimes be suppressed. This disease is believed to be inherited in an autosomal dominant fashion.

A few extra teaching points…

1) Tourette Syndrome is a clinical diagnosis. Diagnostic testing may help you rule out other causes, but it will not directly help you confirm Tourette’s.
2) The diagnostic criteria include the presence of witnessed motor AND phonetic tics before age 21 that are not explained by an underlying medical condition. Tics must occur many times per day, more days than not, for a period of more than one year.
3) Treatment options include dopamine agonists/antagonists, botulinum toxin injection for focal vocal and motor tics, alpha agonists, and SSRIs. The goal of therapy is to reduce the number and frequency of tics in order to improve social interactions at school, work and play. The tics themselves are not harmful.
4) PANDAS, or pediatric autoimmune neuropsychiatric disorder associated with group A streptococci, is a tic disorder that has received much recent attention. It should be excluded in a patient suspected to have Tourette syndrome. There are five diagnostic criteria for PANDAS: pediatric onset, presence of obsessive compulsive disorder and/or a tic disorder, abrupt onset with episodic symptom course, associated with group A strep infections, association with neurological abnormalities like motoric hyperactivity, choreiform movements and tics. Antibiotic therapy tends to result in improvement of the OCD symptoms. Ultimately, however, the exact relationship between PANDAS and Tourette’s is not clear.

Rett syndrome (choice B) is a pervasive developmental disorder that occurs almost exclusively in females. Patients at first demonstrate normal development and then gradually lose speech and purposeful hand use. Austic symptoms, seizures, decreased head growth, stereotypic hand movements, ataxia and breathing difficulties follow. Developmental arrest can begin as early as 6 months of age, but the period of rapid deterioration usually begins between ages 2-4 years old, with the late motor deterioration occurring at around age 10. Because it makes such a distinctive diagnosis (and serves as a nice distractor), Rett syndrome makes frequent appearances on the USMLE.

44
Q

A 51 year old supermarket cashier has had worsening pain and tenderness of her left leg. Her symptoms have been worsening for the past two days, ever since she had to work three consecutive days of “double shifts” due to a co-worker’s absence. Medications include lisinopril for hypertension and hormone replacement therapy for hot flashes. Physical examination shows an obese female with temperature 38.1 C (100.5 F), pulse 88, BP 136/84, and respirations 14/min. There are numerous prominent varicose veins on the legs bilaterally. The left leg has a 6 inch (15 cm) linear area of erythema and edema on the medial aspect of the knee and calf, with a palpable, nodular, cordlike structure located just underneath the skin. Doppler ultrasonography is performed on both legs and shows a single thrombus extending from the middle to distal saphenous vein on the left. Which of the following is the most appropriate next step in the management of this patient’s condition?

A.Rest, elevation, non-steroidal anti-inflammatory agents, and heat

B.Enoxaparin

C.Warfarin

D.Spiral CT of the chest

E.Placement of inferior vena cava filter

A

A.Rest, elevation, NSAIDs, and heat

This patient has superficial thrombophlebitis, suggested by her history and physical and confirmed by ultrasound. Unlike patients with deep venous thrombosis, there is no need for anticoagulation, and treatment is symptomatic with rest, elevation, NSAIDs, and heat (answer A).

A few additional teaching points from this case…

1) In addition to symptomatic treatment of her thrombophlebitis, this patient should also strongly consider discontinuing her hormone replacement therapy (HRT). While HRT is a very effective treatment for menopausal hot flashes, it also leads to a hypercoagulable state and increases the risk of both superficial venous thrombosis (SVT) and DVT.
2) Distinguishing between superficial and deep vein thrombosis is important, because while DVTs can cause pulmonary embolism, SVT does not. Both SVT and DVT can cause swelling, pain, and warmth of the affected extremity, but palpating a “cord” superficially is a giveaway that the thrombosis is truly superficial. (It is true that occasionally DVTs can be palpated, but remember that you are trying to palpate the DEEP veins of the leg – not an easy task in most patients!) Because superficial thrombophlebitis and DVT are caused by the same predisposing factors, it’s always reasonable to get an ultrasound to confirm that there is not also an underlying deep vein thrombosis that could embolize.
3) The saphenous vein is a superficial vein – remember, it is harvested with a simple subcutaneous incision for CABG or vascular grafting! The femoral and popliteal veins are deep veins, so if there is a clot there, you need to start anticoagulation. A common trick is that the superficial femoral vein is still a deep vein, so a clot there requires treatment for DVT.

45
Q

An otherwise healthy 26 year old female presents to her family physician with a rash on her neck. She just noticed it yesterday, but it is pruritic and painful. The patient has not been outdoors, and cannot think of anything in particular that could have caused the rash, other than perhaps using a new hairspray in preparation for her best friend’s wedding several days ago. On physical exam, there is a papular and vesicular erythematous rash in a thin linear pattern that encircles her lower neck. The remainder of her physical exam is normal. What is the most appropriate initial step in the management of this patient?

A.Topical steroid cream

B.Oral prednisone

C.Reassurance

D.IgE testing

E.Oral antibiotics

A

A.Topical steroid cream

This is a classic case of contact dermatitis which can be induced by direct skin contact with an allergen or an irritant. The rash is usually intensely pruritic, and if allergic in nature, can occur up to two weeks after exposure to an offending allergen. The rash is usually papular and erythematous with indistinct margins, distributed along areas of exposure. The immune response causes fluid to collect in the epidermis, which can cause vesicles and oozing. The best treatment for mild or limited contact dermatitis is a medium potency topical steroid (answer A).

46
Q

A 23 year old student has the acute onset of dizziness and nausea and vomiting. He feels like the room is rotating around him to the left. Six days ago, he had a low-grade fever, myalgias, and a sore throat, which resolved yesterday. He denies a history of similar episodes, and his past medical history is entirely benign. Physical examination reveals mild hearing loss on the left and horizontal nystagmus that resolves when the patient looks at a fixed object. Tympanic membranes are clear. Though the patient is able to ambulate, he shows marked gait instability. There is a positive head thrust test. There is no dysarthria, motor weakness, sensory loss, facial droop, or limb dysmetria observed. Which of the following is the most likely diagnosis in this patient?

A.Labyrinthitis

B.Cerebellar infarction

C.Benign paroxysmal positional vertigo

D.Multiple sclerosis

E.Suppurative otitis media

A

A.Labyrinthitis

This patient presents with a single, sudden episode of vertigo following a viral illness. This history, accompanied by the physical examination findings of peripheral nystagmus, hearing loss, and an abnormal head thrust test, strongly point to a diagnosis of acute labyrinthitis (answer A).

Labyrinthitis, also known as vestibular neuritis or vestibular neuronitis, is thought to be caused by postviral inflammation of the vestibular portion of CN VIII. It is a benign, self-limited disorder that completely resolves in most patients after several days – but until it resolves, patients can be quite miserable. Treatment with corticosteroids may shorten the duration of the attack by reducing inflammation, and antiemetics, antihistamines, and benzodiazepines may ameliorate symptoms.

Benign paroxysmal positional vertigo (answer C) causes recurrent episodes of vertigo that last a few seconds – not a few hours, like this patient’s symptoms. They are usually precipitated by specific head movements, and are thought to be caused by the movement of small calcium stones (otoliths) inside the vestibule. The tabletop Dix-Hallpike maneuver will re-create symptoms and confirm the diagnosis if it is in doubt.

47
Q

An otherwise healthy 15 year old male comes to his physician with the complaint of facial acne. He has tried multiple over-the-counter acne washes without improvement, and he and his mother are hopeful that a medication will help. The patient’s diet is acceptable but does include large amounts of fast food. He is under a moderate amount of stress juggling school and his competitive traveling soccer team, but both he and his mom feel that he is handling this well. On physical exam, the patient’s face has scattered open and closed comedones and a few small papules. No scarring is noted, and his back and upper arms are not affected. What is the most appropriate initial step in the management of this patient?

A.Isotretinoin

B.Topical retinoid

C.Oral erythromycin

D.Reassurance

E.Avoidance of greasy foods

A

B.Topical retinoid

For mild comedonal acne, the best treatment is a topical retinoid and/or other topical agents such as salicylic acid, azelaic acid, glycolic acid, and benzoyl peroxide, so out of the choices listed above, a topical retinoid (answer B) is the best option at this point.

Benzoyl peroxide has antimicrobial and comedolytic properties and can be used for non-inflammatory comedonal acne or for more severe inflammatory acne. When combined with a topical antibiotic (like clindamycin) it can help prevent resistant strains of P. acne from emerging.

Oral antibiotics (tetracycline, doxycycline, minocycline, erythromycin, bactrim & clindamycin) are used for inflammatory acne to prevent the growth of P. acnes in the pilosebaceous unit. Oral isotretinoin (answer A) is the treatment for severe acne vulgaris and is not yet indicated in this patient.

48
Q

A 19 year old female comes to her physician because of visual blurring for the past week. For the past few months, she has also has experienced daily headaches, which are worst in the morning upon awakening and improve throughout the day or after taking ibuprofen. Her past medical history is unremarkable, and her only medication is a combination oral contraceptive pill. Physical examination shows height 62 inches (157 cm), weight 206 lbs. (93 kg), temperature 37.2 C (99.0 F), pulse 72, blood pressure 118/76. Fundoscopic examination shows bilateral papilledema. Neurological examination shows no focal deficits. CT shows normal ventricles and no evidence for mass lesion, but increased volume and distension of the perioptic subarachnoid space. Lumbar puncture shows an elevated opening pressure and clear CSF with normal cell counts, protein, and glucose. Which of the following is the most likely diagnosis in this patient?

A.Aseptic meningitis

B.Craniopharyngioma

C.Dandy-Walker malformation

D.Subdural hemorrhage

E.Pseudotumor cerebri

A

E.Pseudotumor cerebri

This is pseudotumor cerebri (answer E), also known as idiopathic intracranial hypertension. Its cause is unknown, but patients – classically obese, young females taking oral contraceptives – develop increased intracranial pressure and papilledema. If untreated, the papilledema can cause optic atrophy and blindness.

Aseptic meningitis (answer A) occurs when a patient has clinical signs and laboratory evidence of meningeal irritation, but negative bacterial cultures. The most common cause is viral meningitis caused by enteroviruses.

49
Q

A 31 year old female presents with several weeks of progressive fatigue, weakness, and weight loss. She has also had occasional nausea and diarrhea. Past medical history includes juvenile diabetes and hypothyroidism. On physical examination, the patient has areas of bronzed hyperpigmentation on her face, neck, and the dorsum of her hands. The remainder of physical exam is within normal limits. Laboratory examination is significant for a sodium of 130 mEq/L and a glucose of 65 mg/dL. Which of the following is the most appropriate next step in the diagnosis of this patient?

A.Thyroid ultrasound

B.Measurement of insulin-like growth factor I (IGF-1)

C.Measurement of 21-hydroxylase activity

D.Cosyntropin stimulation test

E.Dexamethasone suppression test

A

D.Cosyntropin stimulation test

This patient has primary adrenal insufficiency or Addison’s disease. Key features to diagnosis include fatigue, weight loss, hypotension, hyponatremia, and hypoglycemia. To confirm the diagnosis, stimulation with synthetic ACTH or cosyntropin (answer D) should be performed, along with a measurement of plasma cortisol. If the adrenals are functioning, the cortisol should rise with administration of cosyntropin.

This patient’s hyperpigmentation is a classic sign of Addison’s disease. It is caused by excessive ACTH, which stimulates melanocytes. This finding actually helps distinguish between primary and secondary adrenal insufficiency, as well. In a patient who has abruptly stopped a steroid taper, for example, the pituitary has been suppressed by the presence of chronic exogenous glucocorticoids. Thus, ACTH is low, and there should be no hyperpigmentation. Patients whose adrenals are destroyed by an autoimmune process (i.e., primary adrenal insufficiency) have a hyperfunctioning pituitary that is producing lots of ACTH to try to flog the adrenals into making some glucocorticoids, and this excess ACTH stimulates melanocytes as well.

50
Q

A six-month old female is brought to the clinic in December with wheezing and respiratory difficulty for the past several hours. She did have a low-grade fever two days ago accompanied by rhinorrhea, but has otherwise been well since birth. Vital signs are temperature 38.6 C (101.5 F), pulse 160, blood pressure 87/59, respirations 60/min. Nasal flaring and intercostal retractions are noted. Auscultation of the lungs reveals faint inspiratory crackles at the lung bases and diffuse expiratory wheezing. A chest x-ray shows a normal cardiac silhouette, hyperexpanded lungs, and peribronchial thickening. Which of the following tests would be most likely to confirm the diagnosis in this patient?

A.Rigid bronchoscopy

B.Echocardiogram

C.Methacholine challenge test

D.Rapid antigen test for respiratory syncytial virus

E.Sweat chloride test

A

D.Rapid antigen test for respiratory syncytial virus

This child has bronchiolitis, a viral lower respiratory tract infection most often caused by respiratory syncytial virus (answer D).

Over three-fourths of all cases are caused by respiratory syncytial virus (RSV), while most of the rest of the cases are caused by parainfluenza and adenoviruses. 2) A common presenting symptom – especially in infants younger than 6 weeks old – is APNEA, so when you see it, think of RSV infection!

Treatment of RSV infection is largely symptomatic, with humidified oxygen, bronchodilators, and corticosteroids being the mainstays of treatment. Antiviral therapy with ribavirin may be used in children with severe disease. Immunization with palivizumab (Synagis), a monoclonal antibody against RSV, reduces the risk of hospitalization for bronchiolitis, and is given to premature infants or infants with chronic lung or heart disease.

51
Q

A 72 year old female with abdominal pain and unstable vital signs is found on angiography to have a ruptured abdominal aortic aneurysm. Past medical history includes hypertension, diabetes, and chronic renal insufficiency with baseline creatinine of 1.2 mg/dL. The patient is rushed to surgery and her aorta is successfully repaired in an eight hour operation where the patient receives five units of packed red blood cells and several liters of intravenous fluid. On the day following the operation, laboratory analysis reveals plasma creatinine of 2.1 mg/dL and BUN of 46 mg/dL. The fractional excretion of sodium (FeNa) is calculated to be 0.53%, and microscopic urinalysis shows occasional hyaline casts, but no RBCs, WBCs, or cellular or granular casts. Which of the following is the most likely explanation for this patient’s abnormal laboratory values?

A.Cytotoxic effects of iodinated contrast material

B.Reduced blood flow to the kidneys

C.Ischemic necrosis of renal tubule cells

D.Obstruction of the distal urinary outflow system

E.Interstitial inflammation caused by drug hypersensitivity

A

B.Reduced blood flow to the kidneys

This patient presents with acute renal failure (ARF) and a sudden rise in her BUN and creatinine. While she has numerous possible causes for her ARF, the elevated BUN/creatinine ratio, low fractional excretion of sodium, and bland urine sediment all point to pre-renal azotemia, which can be caused by any condition that reduces blood flow to the kidneys (answer B).

52
Q

A 38 year old woman has had four weeks of watery, non-bloody diarrhea and cramping abdominal pain. She denies a history of similar symptoms, sick contacts, drinking unpurified water, and recent travel. There have been no changes to her diet, and she does not drink milk or eat dairy products. Current medications include esomeprazole and ibuprofen. She completed a course of amoxicillin/clavulanate for a sinus infection two months ago. On physical examination, she is afebrile. Bowel sounds are slightly hyperactive, and there is mild lower abdominal tenderness to palpation. Which of the following tests will most likely lead to the diagnosis in this patient?

A.Sudan stain for fecal fat

B.Microscopic examination of the stool for ova and parasites

C.D-xylose test

D.Barium upper gastrointestinal series

E.ELISA for C.difficile-associated toxins A and B

A

E.ELISA for C.difficile-associated toxins A and B

This patient presents with chronic diarrhea (diarrhea lasting >4 weeks), and her only risk factor is recent antibiotic use. The most likely diagnosis in this case is antibiotic-associated diarrhea caused by Clostridium difficile, and the way to confirm this is by an assay for C. difficile toxins A and B (answer E)

53
Q

A 72 year old female presents after “passing out” while walking back to her house from the mailbox. Other than an abrasion on her elbow, she denies any trauma from the fall and recovered quickly following the incident. There was no bowel or bladder incontinence or prodrome before the attack. On review of systems, the patient admits to occasional exertional chest pains that are relieved by taking her husband’s nitroglycerin. Physical examination shows vital signs of heart rate 75 and regular, blood pressure 138/78, and respirations 13/min. There is a small abrasion and associated ecchymosis on the patient’s left elbow, but no other signs of trauma. On cardiac auscultation, there is a harsh, III/VI systolic crescendo-decrescendo murmur heard best at the base of the heart and bilaterally at the clavicles. Lung fields are clear, and there is no peripheral edema. Neurological examination shows no focal abnormality. EKG shows normal sinus rhythm with left ventricular hypertrophy and normal PR and QT intervals. Which of the following is the most appropriate step in the management of this patient’s syncope?

A.Urine drug screen

B.Head CT

C.Tilt table testing

D.Referral to a cardiologist for pacemaker placement

E.Referral to a cardiac surgeon for valve replacement

A

E.Referral to a cardiac surgeon for valve replacement

Given the findings on physical examination, the cause of this patient’s syncope is most likely to be aortic stenosis. The natural history of aortic stenosis includes a long asymptomatic period when their valvular disease does not cause any hemodynamically significant pathology . However, the ultimate course of the disease was illustrated by an old study that demonstrated that once a patient has developed one of three ominous symptoms - angina, syncope, or heart failure - their average survival time is less than two to three years, with a significant risk for sudden death. For this reason, any person with symptomatic aortic stenosis needs referral to a cardiothoracic surgeon for valve replacement (answer E).

54
Q

A 57 year old male presents with three weeks of fatigue. He denies depressed mood, palpitations, changes in bowel habits, syncope, or new medications. On physical examination, the patient’s temperature is 36.7 C (98.6 F), pulse 100, and blood pressure 126/82. Peripheral pulses are 2+, but are noted to be irregularly irregular. Breath sounds are equal bilaterally, and there is no peripheral edema. Initial troponin I is not elevated. The patient is started on intravenous diltiazem, and an EKG is obtained. The patient’s EKG is seen here. Which of the following is the most appropriate next step in the management of this patient?

A.Heparin and warfarin

B.Cardiac catheterization

C.Cardiac defibrillator implantation

D.Ibutilide

E.Electrical conversion

A

A.Heparin and warfarin

This patient presents with what is likely chronic atrial fibrillation. At this point, the most appropriate intervention among those listed is to begin anticoagulation with heparin and warfarin (answer A). This question raises a number of important issues regarding the treatment of atrial fibrillation. 1) When the atrium isn’t squeezing blood out appropriately, thrombus formation can result. If the atrium suddenly starts squeezing again, that thrombus can embolize, causing organ damage or stroke. For this reason, before electrical or pharmacological cardioversion is attempted, the patient should be adequately anticoagulated. (The exception to this is in a severely hemodynamically unstable patient – in that case, the risk of doing nothing is much greater than the risk of causing a stroke.) Even if you’re not going to attempt cardioversion, a patient with either chronic or paroxysmal atrial fibrillation should be anticoagulated to reduce their stroke risk, which is why the best answer among those listed is answer

55
Q

A 25 year old male with a history of intravenous drug abuse presents with fever, chills, and shortness of breath for two days. He last used heroin five days ago. Vital signs include temperature 38.8 C (102 F), pulse 90, blood pressure 116/78, respirations 18/min. There is a grade II/VI systolic murmur heard best at the lower right sternal border. Faint crackles are heard throughout the lung fields. There are no splinter hemorrhages, peripheral petechiae, or erythematous patches on the palms or soles. No abnormalities are seen on fundoscopic exam. Neurological exam is normal. A chest x-ray shows no evidence of pneumonia. Three sets of blood cultures are drawn, and the patient is started on empiric antibiotic therapy with vancomycin and cefepime. Which of the following is the most appropriate next step in the management of this patient?

A.Naloxone

B.Buprenorphine

C.Ciprofloxacin

D.Trans-thoracic echocardiogram

E.D-dimer

A

D.Trans-thoracic echo

Any patient with a heart murmur and a history of i.v. drug use should be considered to have infectious endocarditis until proven otherwise. The best test to pursue that diagnosis is a trans-thoracic echocardiogram (answer D).

56
Q

A 53 year old female with type II diabetes presents is brought to the physician by her husband because of lethargy, fatigue, and decreased responsiveness. She had been feeling well until two days ago, when she developed myalgias and upper respiratory congestion. Medications include glipizide, metformin, lisinopril, and atorvastatin. On physical examination, the patient is somnolent but arousable. Vital signs include temperature 36.7 C (98.6 F), pulse 130, blood pressure 102/64, respirations 15/min. There is no nuchal rigidity. There is poor skin turgor and the patient’s mucous membranes are dry. Laboratory evaluation shows the following: Na+ 124, K+ 4.0, Cl- 92, HCO3- 24, BUN 70, creatinine 2.1, glucose 965, total protein 6.1 g/dL, albumin 4.5 g/dL, AST 12 U/L, ALT 17 U/L, Alk. Phosphatase 110 U/L, total bilirubin 0.7 mg/dL. Which of the following is the most likely diagnosis in this patient?

A.Syndrome of inappropriate antidiuretic hormone secretion (SIADH)

B.Diabetic ketoacidosis

C.Bacterial meningitis

D.Nonketotic hyperosmolar coma

E.Acute sinusitis

A

D.Nonketotic hyperosmolar coma

This diabetic patient presents with lethargy and dehydration following an upper respiratory infection. Given this patient’s lack of an anion gap, the fact that she is a type II diabetic, and the degree of her blood glucose elevation, the most appropriate diagnosis is a hyperosmolar hyperglycemic nonketotic coma or HHNK (answer D).

Some teaching points… 1) In patients with an elevated glucose, it is very important to distinguish between diabetic ketoacidosis (DKA) and nonketotic hyperosmolar states. Some things that help you determine the difference are… a) Type I vs. Type II diabetes – DKA almost always affects type I diabetics. Pathogenetically, in DKA, the body has metabolic fuel available but reacts as if it is starving, because there is an abolute lack of insulin. Patients with type II diabetes usually still make plenty of insulin, but their bodies are resistant to insulin’s actions. If insulin resistance becomes very severe, patients with type II diabetes can develop ketoacidosis, but typically DKA is a complication of type I diabetics. b) Degree of glucose elevation – Patients with DKA very seldom have a glucose that exceeds 700 or 800 mg/dL, while patients with nonketotic hyperglycemia may have a blood glucose that exceeds 1000 mg/dL.

2) The key feature to diagnosis in a patient with DKA is – no surprise – acidosis. This is usually manifested by an increased anion gap. This patient’s anion gap is (Na+) – (Cl-) – (HCO3-) = 124 – 92 – 23 = 9 Although different sources quote different values, the normal range for an anion gap is usually given as 12 +/-4, or from 8 to 16. 3) Both DKA and hyperglycemic hyperosmotic nonketotic coma are commonly precipitated by acute stress (from dehydration, infections, or drug use). Increased stress hormones and glucagon increase the patient’s blood sugar and begin the pathological cascade. 4) This patient’s low sodium is pseudohyponatremia and is caused by her dramatically elevated blood glucose. (The term “pseudohyponatremia” is a bit misleading, because the patient’s sodium really is decreased.) Pseudohyponatremia occurs whenever there is a high concentration of glucose, triglycerides, or ketones in the blood. Though the relationship between glucose increase and sodium decrease is nonlinear, a useful rule of thumb is that the sodium concentration will drop 2.5 mEq/L for every 100 mg/dL of glucose rise above normal. SIADH (answer A) should always be considered when you encounter a patient with hyponatremia (especially one on the boards). However, as described above, this patient’s hyponatremia is a direct consequence of her extremely high glucose. As described above, patients with diabetic ketoacidosis (answer B) have an elevated anion gap. Bacterial meningitis (answer C) should be considered and ruled out in any patient that presents with lethargy. However, given this patient’s lab findings, it is not the most likely diagnosis. Patients with acute sinusitis (answer E) complain of nasal congestion, fever, and headache, often with pain that radiates to the upper teeth. Treatment is with decongestants and antibiotics like amoxicillin or TMP/SMZ.

57
Q

A 52 year old female comes in for a yearly physical. She has been feeling completely well, and denies chest pain, shortness of breath, weight loss, decreased exercise tolerance, or change in stool frequency or caliber. Past medical history is significant for hypertension controlled with hydrochlorothiazide. Although she has a history of several abnormal Pap smears thirty years ago, her tests have been normal since then, with her last normal Pap occurring two years ago. She had a normal mammogram earlier this year. She has smoked one half a pack of cigarettes daily for thirty-five years, and drinks 1-2 glasses of wine nightly. Her mother died at age 63 from ovarian cancer, and her father died at age 70 from lung cancer. Her siblings are alive and well. EKG shows sinus rhythm with a normal axis and intervals. Of the following, which is the most appropriate screening test to offer this patient?

A.Pap smear

B.CA-125

C.Chest x-ray

D.Exercise stress test

E.Colonoscopy

A

E.Colonoscopy

An exercise stress test (answer D) is used to risk stratify patients who have a moderate pre-test probability of having coronary artery disease. Although this patient does smoke and has hypertension, she is still fairly young, has a normal EKG, no significant family history of CAD, and no symptoms that suggest CAD. At this point, her pre-test probability of having significant coronary artery disease is probably fairly low, and a stress test is not needed.

58
Q

An 18 month old male presents for a well-child examination. While observing the child playing in his mother’s lap, it appears that the child has a wandering right eye. The patient’s mother denies any family history of visual disorders or cancer, and says that the child has appeared completely healthy lately. On physical examination, there is no tenderness to mild pressure on either eye. Sclera are non-injected. A white pupillary reflex is present on the right. With the child visually fixed on a toy, each of his eyes are separately covered and then uncovered. There is no eye movement when the patient’s right eye is covered and rapidly uncovered, but the patient’s right eye refixates on the toy when the left eye is covered and then uncovered. What is the most appropriate next step in the management of this patient?

A. MRI of head

B. Occlusion of left eye with patch

C. Occlusion of right eye with patch

D. Atropine eye drops

E. Reassurance

A

A. MRI of head

This child presents with leukocoria, which is the hallmark physical finding of retinoblastoma. This is a very serious and life-threatening disease that requires immediate diagnosis with CT or MRI (answer A) and ophthalmologic referral. Ultimately, if the patient has retinoblastoma, he will require urgent surgery, since the tumor tends to metastasize early. Light normally transmits easily through the eye, and any anatomical opacity will cause a dulling of the normal red reflex to a grey or white color. Thus, disorders of the lens (like cataracts) or vitreous (like hemorrhage) can cause leukocoria – but it is also the most frequent presenting physical finding for retinal disorders like retinoblastoma, and these must be ruled out first.

A commonly-tested fact about retinoblastoma is its heritability: 40% of patients have mutations to both RB tumor suppressor genes. These patients may have a family history of retinoblastoma, and they are also at risk for developing bilateral disease. However, the majority of patients with retinoblastoma – 60% - have sporadic mutations that cause their illness.

59
Q

A four year old male presents with a painful right hand. One week ago, he was bitten by his pet kitten on the hand. His mother also reports an occasional low-grade fever. On physical exam, there are several erythematous, painful cutaneous papules and pustules present near the nearly healed bite site, as well as tender unilateral axillary lymphadenopathy on the right. Which of the following is the organism most likely to be the cause of this patient’s infection?

A. Methicillin-sensitive Staphylococcus aureus

B. Yersinia pestis

C. Francisella tularensis

D. Bartonella henselae

E. Pasteurella multocida

A

D.Bartonella henselae

This is “cat scratch disease,” caused by Bartonella henselae. Antibiotics like TMP/SMZ or azithromycin may shorten symptom duration by a day or two in a normal patient, and should be used in any immunocompromised patient due to the increased risk of developing disseminated disease.

Pasteurella multocida (answer E) is found in the mouths of animals like cats and dogs and causes cellulitis after a bite. The unusual feature is that the cellulitis develops very quickly after the bite – often within just a few hours.

60
Q

A 19 year old male presents to the emergency room at midnight with wheezing and shortness of breath. Although history is difficult to obtain secondary to the patient’s tachypnea, he is able to state that his symptoms started one hour ago and have been progressively worsening. Past medical history is significant for seasonal allergies and asthma. Current medications include inhaled albuterol and budesonide. On physical exam, the patient appears anxious and is sitting upright. Vital signs on presentation are pulse 120, respirations 42/minute, BP 118/68, and O2 Sat 93% on room air. Nasal flaring is noted. The skin appears pale. No murmurs, rubs, or gallops are appreciated on cardiac auscultation. Bilateral, symmetrical wheezing is present. ABG obtained on presentation shows pH 7.51, pO2 70 mm Hg, and pCO2 29 mm Hg. Supplemental O2 by nasal cannula is started, and the patient is treated with nebulized albuterol and i.v. methylprednisolone. Thirty minutes after treatment, the patient is reassessed. No wheezing is noted, and breath sounds are equal bilaterally. Vitals are pulse 121, respiratory rate 35/minute, BP is 120/71, which drops to 108/65 on inspiration. Repeat ABG is obtained, showing pH 7.40, pCO2 41 mm Hg, pO2 72 mm Hg. What is the most appropriate next step in management?

A.Obtain chest x-ray

B.Begin i.v. albuterol

C.Begin i.v. theophylline

D.Administer inhaled salmeterol

E.Intubation and mechanical ventilation

A

E.Intubation and mechanical ventilation

Key teaching point: A rising pCO2 in an asthma attack signals impending respiratory failure, not improvement! It is important to recognize that this patient is getting worse, not better. Typically, in an acute asthma attack, the patient’s tachypnea causes them to “blow off” CO2, resulting in a primary, uncompensated respiratory alkalosis. Rising CO2 in the face of sustained tachypnea is a very ominous sign - it shows that the patient’s airways are so constricted that he is no longer able to get rid of CO2! If a normal patient began breathing at 35-40/minute (like this patient) they would very soon have a pCO2 in the teens or 20s as they blew off their CO2. This patient also appears to be fatiguing as well - his respiratory rate has dropped from 42 to 35. Therefore, he must be intubated and mechanically ventilated to prevent hypercarbia and respiratory failure (answer E).

Besides the ABG findings, there are several other key clues in the question stem that show you that this patient’s asthma did not improve with initial therapy. The presence of pulsus paradoxus (a drop of >10 mm Hg on inspiration) correlates directly with the severity of the attack and is a predictor of a PCO2 >35mm Hg. Additionally, the patient’s absent wheezing does not represent improvement, but rather the dreaded “silent chest” of a severe asthma attack. Remember that in order to hear wheezing on auscultation, you have to be moving enough air to create a sound! This patient has poor air flow, not clear lungs.

The initial treatment for this patient was completely appropriate. Management of the acute asthma exacerbation begins - like any other situation - with the ABCs (airway, breathing, circulation). Start an i.v. and begin supplemental O2. Then give B2 agonists like albuterol. In an acute situation, a nebulized dose of albuterol is usually preferable to an inhaler, since patients may be frightened and unable to adequately time and coordinate the use of an inhaler.

61
Q

A 13 month old female presents to her doctor two weeks after being successfully treated for an uncomplicated febrile UTI. Since that time, a voiding cystourethrogram (VCUG) showed grade I reflux bilaterally, and a renal ultrasound showed normal kidneys. The patient’s past medical history is otherwise unremarkable. On physical exam, the patient is afebrile. Urinalysis is negative for leukocyte esterase and nitrite and microscopic examination shows only occasional hyaline casts and squamous epithelial cells. What is the most appropriate next step in the management of this patient?

A.Culture urine monthly

B.Repeat renal ultrasound in one month

C.Begin antibiotic prophylaxis

D.Repeat VCUG in three months

E.Reassurance

A

C. Begin abx ppx

Current recommendations for grade I-II reflux include medical management due to the high likelihood that the reflux will resolve on its own as the child grows older. Low dose antibiotic therapy (answer C) with TMP/SMZ or nitrofurantoin is appropriate until you document that the VUR has disappeared. Until then, routine surveillance cultures should be obtained every 3-4 months and whenever the child has an unexplained fever or symptoms of a UTI.

62
Q

A 65 year old male comes to the physician because of fatigue and increasing abdominal girth. Over the past six months, he has also experienced an unintentional weight loss of 15 lbs., which he believes is due to the fact that he feels full after only a few bites of his meals. The patient has no past medical history, as this his first visit to a physician since being discharged from the army 40 years ago. The patient denies smoking but does drink 1-3 beers every evening. On physical examination, the patient appears emaciated with a visibly distended and protuberant abdomen. No icterus is noted. Neck veins are non-distended and lungs are clear to auscultation. There is an S4 present on cardiac auscultation. Abdominal examination shows shifting dullness to percussion and a positive fluid wave. There is a firm, red, nontender nodule at the umbilicus. Examination of the skin shows no spider angiomata or palmar erythema. There is 1+ edema at the ankles bilaterally. Labaratory results include albumin of 3.6 g/dL, AST 39 U/L, and ALT 29 U/L. Paracentesis is performed, yielding clear, straw colored fluid, with an albumin concentration of 2.7 g/dL, 650 leukocytes (85% lymphocytes, 15% PMNs). No organisms are seen on Gram stain. Which of the following is the most likely cause of this patient’s ascites?

A.Spontaneous bacterial peritonitis

B.Alcoholic cirrhosis

C.Budd-Chiari syndrome

D.Congestive heart failure

E.Peritoneal metastatic disease

A

E.Peritoneal metastatic disease

However, it is now clear that you can classify ascites with 95% accuracy only by knowing the serum albumin and the ascites albumin concentration and calculating the serum-ascites albumin gradient or SAAG. If the difference between the serum albumin and the ascites albumin is greater than 1.1, then the ascites is caused by portal hypertension – with cirrhosis, right sided CHF, and Budd-Chiari syndrome being the most frequent culprits. If the SAAG is less than 1.1, then the ascites is NOT caused by portal hypertension. In this situation, the most likely causes include pancreatitis, peritonitis, and peritoneal carcinomatosis. This patient’s serum albumin is 3.6, while his ascites has an albumin of 2.7. This gives him a SAAG of 0.9, and indicates that his ascites is more of the exudative variety. Among the possibilities listed, the only one that causes a low-SAAG ascites is peritoneal metastatic disease (answer E). This patient also has a finding that suggest where his primary cancer might be: the firm nodule at the umbilicus is a Sister Mary Joseph’s node, which is the calling card of gastric carcinoma.

63
Q

A 19 year old college female presents to student health because she is no longer menstruating. She has missed her last 5 menses and states that the 3 prior menses were very light and unpredictable. Menarche occurred at age 10 and her menses had been occurring at regular at 24 day intervals until this recent change. She is sexually active using only condoms for birth control. She takes no medications. The patient denies breast tenderness, abdominal fullness, and nausea, though she has noticed a 10 pound weight gain over the past few months. She feels that she might have a little less energy than usual. Physical exam is within normal limits. BMI is 21. A qualitative serum beta-hCG is negative for pregnancy. What is the most appropriate next step in the management of this patient?

A.Urine pregnancy test

B.Quantitative serum beta-hCG

C.Follicle stimulating hormone (FSH)

D.Serum thyroid stimulating hormone (TSH) and prolactin

E.Serum androgen levels

A

D.Serum thyroid stimulating hormone (TSH) and prolactin

The first step in the secondary amenorrhea work-up is serum is to rule out pregnancy. Serum beta-hCG is more sensitive test than the home pregnancy test and should be done even if the patient has had a negative home pregnancy test. For the young female, the most common causes of secondary amenorrhea besides pregnancy are thyroid and pituitary dysfunction, so checking a serum TSH and prolactin is the next step (answer D) In addition, the patient in the question stem has had weight gain and a possible change in energy, so hypothyroidism is something in particular to be thinking about in your work-up.

64
Q

A 10 year old male presents with a one week history of a limp that seems to be getting worse. The patient describes hip and upper leg pain that is worst when he stands with his full weight on his left foot. The pain started last week, but yesterday it suddenly worsened after he jumped off of a moving swing yesterday. On physical examination, the patient moderately obese and at Tanner stage 2 of sexual development. He walks with a pronounced limp, and refuses to bear weight on the left foot. There is marked limitation of both active and passive range of motion at the hip. Examination of other joints is within normal limits. Which of the following is the most likely diagnosis in this patient?

A.Legg-Calve-Perthes disease

B.Slipped capital femoral epiphysis

C.Transient synovitis

D.Osgood-Schlatter disease

E.Growing pains

A

B.Slipped capital femoral epiphysis

This is a fairly classic presentation of SCFE, or slipped capital femoral epiphysis (answer B). This is a common cause of hip pain and altered gait in obese pre-adolescents, and is caused by displacement of the capital femoral epiphysis from the femoral neck through the growth plate.

A few extra teaching points… 1) The plain film x-rays for this patient would show posterior displacement of the left femoral epiphysis. This finding is often described as “ice-cream slipping off a cone.” 2) It is important to recognize this diagnosis and treat it in a timely manner to prevent joint damage. Patients should be immediately referred to an orthopedic surgeon for the placement of a screw through the center of the epiphysis.

65
Q

A 38 year old woman has sudden onset of nausea and vomiting and severe, colicky flank pain that radiates to the perineum. On physical examination, the patient shifts position frequently and seems unable to sit still. Urinalysis shows trace protein, 3+ blood, pH = 7.2. Microscopic analysis shows many red cells and a few white cells, but no casts or bacteria. Laboratory analysis shows: Na+ 141 K+ 3.0, Cl- 119, HCO3- 12, BUN 17, Creatinine 1.0, glucose 111, calcium 9.8, magnesium 1.6, phosphate 3.0. Which of the following is the most likely etiology of this patient’s electrolyte abnormalities?

A.Increased production of endogenous lactic acid

B.Decreased net secretion of H+ at the distal renal tubule

C.Deficiency of aldosterone

D.Excessive release of parathyroid hormone (PTH)

E.Physiological stress response to pain

A

B.Decreased net secretion of H+ at the distal renal tubule

Key teaching point: the differential diagnosis for a NON-anion gap metabolic acidosis is short – think of RTAs and GI bicarbonate loss. This question describes a patient with nephrolithiasis who is subsequently found to have a non-anion gap metabolic acidosis. These findings are consistent with a type 1 renal tubular acidosis (RTA) in which there is impaired secretion of H+ into the distal tubule and collecting duct (answer B). This in turn leads to the decreased bicarbonate and metabolic acidosis, and the alkalinization of the urine causes nephrolithiasis. Other key findings that help you make the diagnosis of a Type 1 or distal RTA are the low K+ and urine pH>5.5.

66
Q

A 25 year old graduate student complains of a “lump” under his clavicle for the past two weeks. He first noticed the lump after being hit in the shoulder during a rugby match. He has otherwise been well, and denies any other symptoms including fever, night sweats, weight loss, or recent infections or illnesses. On physical examination, the patient is comfortable and afebrile. There is a 4 cm, rubbery, nontender supraclavicular lymph node on the left, as well as numerous 1-2 cm enlarged cervical and axillary nodes. The clavicle is nontender, and there is full range of motion at both shoulders. There is no evidence of a draining abscess or resolving cellulitis. Laboratory evaluation shows the following. WBC 8.0 k/mcL (Normal: 4.5-11.0 k/mcL) Differential: 66% neutrophils, 30% lymphocytes, 3% monocytes, 1% eosinophils Hemoglobin 14.3 g/dL (Normal: 13.5-17.5 g/dL) Hematocrit 43% (Normal: 39-49%) Platelets 212 k/mcL (Normal: 150-450 k/mcL) Which of the following is the most likely diagnosis in this patient?

A.Acute myelogenous leukemia

B.Infectious mononucleosis

C.Hodgkin’s lymphoma

D.Trauma

E.Felty’s syndrome

A

C.HL

This description of a young, asymptomatic patient with cervical lymphadenopathy is typical for a patient with Hodgkin’s lymphoma (answer C). Some teaching points… -Most patients with Hodgkin’s lymphoma present with asymptomatic lymphadenopathy (70%) or an incidentally-found widened mediastinum. The presence of symptoms generally indicates a worse prognosis. Typical symptoms include the “B symptoms” of fever, night sweats, and unexplained weight loss. More rarely, patients complain of pruritus (especially after a hot shower) or severe pain following alcohol ingestion. If any of these findings are described on the USMLE, it’s almost always a tipoff that the diagnosis is Hodgkin’s. -There are several commonly used staging systems for Hodgkin’s lymphoma, but the only one that you’re likely to be tested on is the simple Ann Arbor system. Under this classification, stage I disease involves only a single lymph node region, stage II disease involves two or more lymph node regions on the same side of the diaphragm, stage III disease involves lymph node regions on both sides of the diaphragm, and stage IV describes disseminated disease. In addition to the stage, the designations “A” and “B” are used to describe the absence (A) or presence (B) of the “B symptoms” described above (so that a patient might be described as having stage II B or stage III A disease). -Demographically, Hodgkin’s disease has a bimodal age distribution, with one peak in the 20s-30s and another after age 50. It is one of a relatively small number of cancers that affects otherwise healthy young adults.

Acute myelogenous leukemia (answer A) usually presents with symptoms related to pancytopenia (like easy bruising, anemia, or neutropenia). The CBC should show a high WBC, and microscopic examination should show numerous circulating myeloid blasts. If you suspect this diagnosis, the next step is a bone marrow biopsy.

Felty’s syndrome (answer E) is one of the USMLE’s favorite eponyms. It consists of the triad of seropositive rheumatoid arthritis, splenomegaly, and granulocytopenia.

67
Q

A 38 year old female presents with weakness and tingling in her hands and lower extremities. She was feeling well until yesterday when she noted a “pins-and-needles” sensation in her fingers and toes. She noted weakness in her hands and legs today, and had difficulty holding a pen or walking without dragging her toes on the ground while at work today. Her past medical history includes gastroesophageal reflux and an overnight hospitalization three weeks ago for dehydration following an episode of gastroenteritis. Physical examination shows poor inspiratory effort and diminished breath sounds bilaterally. Deep tendon reflexes cannot be elicited at the ankle, knee, wrist, or elbow, and the patient has 3+ muscle strength distally and 4+ muscle strength proximally. There is a flexor plantar response. Cerebellar testing and cranial nerve testing show no gross deficit. Which of the following is the most likely diagnosis in this patient?

A.Cauda equina syndrome

B.Multiple sclerosis

C.Amyotrophic lateral sclerosis

D.Myasthenia gravis

E.Acute inflammatory demyelinating polyneuropathy

A

E.Acute inflammatory demyelinating polyneuropathy

This patient presents with ascending paralysis and loss of deep tendon reflexes following a self-limited gastrointestinal illness a couple of weeks ago. This is a classic presentation of acute inflammatory demyelinating polyneuropathy (answer E), also known as Guillain-Barre syndrome

The absolutely hallmark sign of Guillain-Barre is the lack of deep tendon reflexes on physical exam.

Treatment is primarily supportive, but plasma exchange and intravenous immunoglobulin (IVIg) shorten the course of the illness. In general, the disease is self-limiting, and most patients fully recover within a few weeks. 4) The most feared complication of Guillain-Barre is respiratory compromise, which may require intubation and mechanical ventilation. The most important thing that you can do for a patient with Guillain-Barre is monitor their pulmonary function and intervene as necessary.

68
Q

At the insistence of his wife, a 71 year old farmer presents to have a “bump” on his shoulder evaluated. The bump has been there for several years, and though it is not painful, it does itch from time to time. The patient does report many years of sun exposure while working on his farm. Physical examination reveals a comfortable, fair-skinned gentleman. On the patient’s left shoulder, there is a smooth, 3cm by 4cm papular lesion with numerous dilated blood vessels throughout. A photo of this lesion is seen here. Which of the following is the most appropriate therapy for this patient?

A.Podophillin resin

B.Shave biopsy

C.Local excision

D.Topical tacrolimus

E.Excision, lymph node biopsy, and radiation

A

C.Local excision

This is a basal cell carcinoma, the single most common cancer in humans. Note the smooth, pearly appearance and multiple telangiectasias. Fortunately, basal cell carcinomas rarely metastasize. However, they can grow and become locally destructive (especially in a cosmetically-sensitive location such as the face.) For that reason, the gold standard of therapy is removal of the tumor through some type of local therapy such as cryosurgery, local radiation, topical 5-fluorouracil, or surgical excision (answer C).

69
Q

A 10 year old boy returns to the physician for a non-healing rash on his arms, legs, and face. Two weeks ago, he initially developed a small, intensely pruritic area of erythema on his arm that progressed into small fluid-filled blisters. He was initially treated with prednisone for contact dermatitis, but after the rash continued to spread, he was given a prescription for cephalexin. Despite finishing the course of cephalexin, the rash has continued to spread to his legs, neck, and face. The patient’s immunizations are up-to-date, and he has never been hospitalized and takes no chronic medications. On physical examination, the child is comfortable and pleasant, with vital signs including temperature 36.8 C (98.2 F), pulse of 90/min, blood pressure 100/68, and oxygen saturation of 99% on room air. There are numerous linear areas of erythema and excoriation are noted on the patient’s arms, legs, neck, and face. The lesions appear to be in various stages of evolution: some are vesicular, while some are covered with a golden-yellow crust. Gram stain of one of the lesions shows numerous Gram-positive cocci in clumps. Which of the following is the most appropriate treatment for this patient at this time?

A.Trimethoprim-sulfamethoxazole

B.Acyclovir

C.Dicloxacillin

D.Vancomycin

E.Varicella zoster immune globulin (ZVIG)

A

A.TMP-SMX

This patient’s history and physical exam findings are classic for impetigo, and the finding of S.aureus on Gram stain merely confirms your clinical suspicion. However, this patient’s infection did not respond to what would seem to be appropriate therapy with cephalosporins. This very strongly suggests that the infection is caused by a resistant strain of S.aureus – most likely community-acquired methicillin-resistant Staphylococcus aureus, or CA-MRSA. CA-MRSA is an increasing problem that has attracted a great deal of media attention lately. Always suspect this diagnosis when there is a history of treatment failure with appropriate beta-lactam or cephalosporin antibiotics. Unlike hospital-associated MRSA, which is generally susceptible only to high-powered antibiotics like vancomycin and linezolid, CA-MRSA strains (fortunately) usually retain susceptibility to clindamycin and trimethoprim-sulfamethoxazole (answer A). Mupirocin ointment is also effective if the patient has only a small patch of impetigo, but topical treatment is impractical for patients with larger areas of infection.

70
Q

A 46 year old homeless man is brought to the hospital after passersby witnessed him having convulsions on the street. On physical examination, the patient is disheveled, unshaven, and has a faint odor of alcohol on his breath. He is semicomatose, and arouses only to pain. Vital signs are temperature 38.3 C (101 F), pulse 115, blood pressure 166/96, respirations 15/min, and oxygen saturation 98% on room air. There is a 4cm simple laceration on his left temple. There are no other visible injuries, and the remainder of the physical examination is normal. Laboratory evaluation shows the following: Hematocrit 38%; mean corpuscular volume 109 fL; sodium 133 mEq/L; glucose 62 mg/dL; magnesium 1.0 mEq/L; albumin 3.0 g/dL; ALT 60 IU/L; AST 123 IU/L; PT 19 s. The patient is placed in mechanical restraints and i.v. fluid with glucose, thiamine, folate, and magnesium is begun. Which of the following is the most appropriate next step in the management of this patient?

A. Intravenous diazepam

B. Oral phenobarbital

C. MRI of head and neck

D. Lumbar puncture

E. Intravenous somatostatin

A

This patient presents with acute alcohol withdrawal. The seizure that he suffered on the street was likely a withdrawal seizure, and given his altered vital signs, he is likely experiencing delirium tremens as well. Before completing any additional diagnostic workup, the patient needs anticonvulsant therapy, and the best choice listed is i.v. diazepam (answer A).

key lab abnormalities seen in alcoholics. These include hypoglycemia; macrocytic anemia; elevated AST and ALT, with a 2:1 ratio of AST/ALT; low magnesium; elevated PT; and low albumin.

71
Q

A 31 year old female presents to her physician in January with several days of fever, malaise, and a cough productive of yellow-green sputum. She had been feeling well until four days ago when she developed a sore throat and rhinorrhea; this was soon followed by fever, malaise, and myalgias. Two days ago she began experiencing a hacking cough that has been occasionally productive of purulent sputum. Vital signs include temperature 37.2 C (99.0 F), pulse 80, blood pressure 118/78, respirations 14/min, oxygen saturation 98% on room air. No crackles or rhonchi are appreciated on auscultation of the lungs. Which of the following is the most appropriate initial step in the management of this patient?

A. Azithromycin

B. Ceftriaxone and levofloxacin

C. Salmeterol

D. Amantadine

E. Pseudophedrine and acetaminophen

A

E.Pseudophedrine and acetaminophen

This patient has acute bronchitis, which is inflammation of the tracheobronchial tree that is almost always caused by viruses. Thus, treatment is symptomatic, and agents like pseudophedrine and acetaminophen (answer E) may help the patient feel a little better.

Some extra teaching points… -Acute bronchitis is almost always caused by viruses. This explains why multiple studies have shown that patients with acute bronchitis do not benefit from antibiotics. Chronic bronchitis, however, is a different story. Patients with COPD and chronic bronchitis who have an exacerbation SHOULD be empirically treated with antibiotics, because studies have shown a benefit to doing so.

-Many patients (and unfortunately many physicians, too) mistakenly believe that yellow or green sputum is an absolute sign of bacterial infection and mandates antibiotic treatment. The yellow-green coloration comes from the myeloperoxidase of neutrophils, so discolored sputum only means that neutrophils are activated – nothing less, nothing more. While neutrophils are the cornerstone of the body’s immune response to bacteria, they also mount the initial response to ANY infectious agent, including viruses.

The key feature is a cough productive of purulent sputum in the setting of other features suggestive of a viral upper respiratory infection.

-Obviously, you don’t want to miss a diagnosis of pneumonia in a patient who presents with cough and purulent sputum. If a patient has abnormal vital signs or crackles on physical exam, you should order a chest x-ray, but in the absence of these signs, it is unlikely to change your management plan.

Azithromycin (answer A) is jokingly referred to by one UVA attending as “our most effective broad-spectrum antiviral,” a reference to the fact that so many practicioners prescribe a Z-Pak for the common cold or other self-limited viral infections. However, azithromycin and other macrolides are very useful in the treatment of community-acquired pneumonia because they can target atypical causes of pneumonia (like Mycoplasma) that are insensitive to beta-lactams.

Ceftriaxone and levofloxacin (answer B) is an antibiotic regimen that might be used in a hospitalized patient with community-acquired pneumonia.

72
Q

A 43 year old teacher returns to her physician after routine lab evaluation revealed elevated liver transaminases with an ALT of 158 U/L and an AST of 108 U/L. The patient has been in her normal state of health, and has no complaints. Past medical history is significant for diabetes mellitus and hypertension, and current medications include metformin and enalapril. The patient denies foreign travel, blood transfusions, or use of over-the-counter or herbal medications. She is a Jehovah’s Witness and does not smoke or use alcohol. Physical examination shows a comfortable, obese female. There is no organomegaly, right upper quadrant or abdominal tenderness, jaundice, scleral icterus, asterixis, or spider angiomata. Laboratory evaluation shows the following: Hepatitis A IgM: negative; Hepatitis A IgG: positive; Hepatitis B surface antigen (HBsAg): negative; Hepatitis B surface antibody (HBsAb): positive; Hepatitis B core antibody (HBcAb): negative; Hepatitis C PCR: negative. Which of the following is the most likely explanation for this patient’s elevated liver enzymes?

A. Hepatitis A infection

B. Hepatitis B infection

C. Adverse effect of metformin

D. Fatty infiltration of the liver

E. Alcohol abuse

A

D. Fatty infiltration of the liver

This is a common problem in outpatient medicine: elevated liver enzymes in an asymptomatic patient. Adverse medication effects, infections, autoimmune diseases, and alcohol abuse can all cause elevated LFTs, but those causes are not likely in this patient. In an overweight patient with diabetes and elevated LFTs, the most likely cause – once other things have been ruled out – is very likely to be fatty infiltration of the liver (answer D) or non-alcoholic steatohepatitis (NASH).